Sample Problem 1/1 Sample Problem 1/3: Solution

You might also like

Download as pdf or txt
Download as pdf or txt
You are on page 1of 16

Article 1/9 Chapter Review 19 20 Chapter 1 Introduction to Statics

SAMPLE PROBLEM 1 /1 SAMPLE PROB LEM 1 /3 y


m = 1400 kg
Determine the weight in newtons of a car whose mass is 1400 kg. Convert For the vectors V1 and V2 shown in the figure,
the mass of the car to slugs and then determine its weight in pounds. V1 = 4 units
(a) determine the magnitude S of their vector sum S V1 V2
(b) determine the angle between S and the positive x-axis
Solution. From relationship 1/3, we have
(c) write S as a vector in terms of the unit vectors i and j and then write a unit j 45°
W mg 1400(9.81) 13 730 N Ans. vector n along the vector sum S i x
Helpful Hints 30°
From the table of conversion factors inside the front cover of the textbook, we (d) determine the vector difference D V1 V2
see that 1 slug is equal to 14.594 kg. Thus, the mass of the car in slugs is Our calculator indicates a result of V2 = 3 units
13 734 N. Using the rules of signifi-
1 slug cant-figure display used in this text- y
m 1400 kg 95.9 slugs Ans. Solution (a) We construct to scale the parallelogram shown in Fig. a for adding
14.594 kg book, we round the written result to
four significant figures, or 13 730 N. V1 and V2. Using the law of cosines, we have
V1 = 4 units
Finally, its weight in pounds is Had the number begun with any
S2 32 42 2(3)(4) cos 105
W mg (95.9)(32.2) 3090 lb Ans.
digit other than 1, we would have 45° 60°
rounded to three significant figures. 45° S
S 5.59 units Ans.
As another route to the last result, we can convert from kg to lbm. Again using α x
A good practice with unit conversion (b) Using the law of sines for the lower triangle, we have 30°
the table inside the front cover, we have 105°
is to multiply by a factor such as
1 slug sin 105 sin( 30 ) V2 = 3 units
1 lbm , which has a value of 1,
m 1400 kg 3090 lbm 14.594 kg
0.45359 kg 5.59 4
because the numerator and the de- (a)
nominator are equivalent. Make sure sin( 30 ) 0.692
The weight in pounds associated with the mass of 3090 lbm is 3090 lb, as calcu- D
lated above. We recall that 1 lbm is the amount of mass which under standard that cancellation of the units leaves
( 30 ) 43.8 13.76 Ans.
conditions has a weight of 1 lb of force. We rarely refer to the U.S. mass unit lbm the units desired; here the units of
y V1
in this textbook series, but rather use the slug for mass. The sole use of slug, kg cancel, leaving the desired units
(c) With knowledge of both S and , we can write the vector S as
rather than the unnecessary use of two units for mass, will prove to be powerful of slug. –V2
and simple—especially in dynamics.
S S[i cos j sin ]
x
Note that we are using a previously calculated result (95.9 slugs). We must be sure that when a calculated number is 5.59[i cos 13.76 j sin 13.76 ] 5.43i 1.328j units Ans.
needed in subsequent calculations, it is retained in the calculator to its full accuracy, (95.929834 . . .) until it is needed.
This may require storing it in a register upon its initial calculation and recalling it later. We must not merely punch 95.9 S 5.43i 1.328j V2
into our calculator and proceed to multiply by 32.2—this practice will result in loss of numerical accuracy. Some Then n 0.971i 0.238j Ans.
S 5.59
individuals like to place a small indication of the storage register used in the right margin of the work paper, directly (b)
beside the number stored. (d) The vector difference D is
Helpful Hints
D V1 V2 4(i cos 45 j sin 45 ) 3(i cos 30 j sin 30 )
SAMPLE PROB LEM 1 /2 You will frequently use the laws of
m = 70 kg
0.230i 4.33j units Ans. cosines and sines in mechanics. See
Use Newton’s law of universal gravitation to calculate the weight of a 70-kg
person standing on the surface of the earth. Then repeat the calculation by using Art. C/6 of Appendix C for a review of
The vector D is shown in Fig. b as D V1 ( V2). these important geometric principles.
W mg and compare your two results. Use Table D/2 as needed.

Solution. The two results are A unit vector may always be formed
by dividing a vector by its magnitude.
Gmem (6.673 10 11
)(5.976 1024)(70) Note that a unit vector is dimen-
W 688 N Ans.
R2 [6371 103]
2
sionless.
R me
W mg 70(9.81) 687 N Ans.

The discrepancy is due to the fact that Newton’s universal gravitational law does
Helpful Hint
not take into account the rotation of the earth. On the other hand, the value g
9.81 m/s2 used in the second equation does account for the earth’s rotation. Note The effective distance between the
that had we used the more accurate value g 9.80665 m/s2 (which likewise ac- mass centers of the two bodies in-
counts for the earth’s rotation) in the second equation, the discrepancy would volved is the radius of the earth.
have been larger (686 N would have been the result).

Article 2/3 Rectangular Components 31 Article 2/4 Moment 41

SAMPLE PROBLEM 2/3 y SAMPLE PROB LEM 2/5 2m


The 500-N force F is applied to the vertical pole as shown. (1) Write F in j Calculate the magnitude of the moment about the base point O of the 600-N A
terms of the unit vectors i and j and identify both its vector and scalar compo- force in five different ways. 40°
nents. (2) Determine the scalar components of the force vector F along the y′
j′
x - and y -axes. (3) Determine the scalar components of F along the x- and y -axes. A 600 N
30° 4m
x
i Solution. (I) The moment arm to the 600-N force is

Solution. Part (1). From Fig. a we may write F as F = 500 N d 4 cos 40 2 sin 40 4.35 m O
30°
F (F cos )i (F sin )j i′ 2m
By M Fd the moment is clockwise and has the magnitude
(500 cos 60 )i (500 sin 60 )j 40°
x′
MO 600(4.35) 2610 N m Ans.
(250i 433j) N Ans. 4m
y y′ 600 N
(II) Replace the force by its rectangular components at A, 40° d
The scalar components are Fx 250 N and Fy 433 N. The vector compo- Fx
nents are Fx 250i N and Fy 433j N. A x A j′
θ = 60° F1 600 cos 40 460 N, F2 600 sin 40 386 N
Part (2). From Fig. b we may write F as F 500i N, so that the required O
scalar components are F
Fy F By Varignon’s theorem, the moment becomes 2m F1 = 600 cos 40°
Fx 500 N Fy 0 Ans. i′ MO 460(4) 386(2) 2610 N m Ans.
Part (3). The components of F in the x- and y -directions are nonrectan- x′
4m
(a) (b) (III) By the principle of transmissibility, move the 600-N force along its F2 = 600 sin 40°
gular and are obtained by completing the parallelogram as shown in Fig. c. The
magnitudes of the components may be calculated by the law of sines. Thus, line of action to point B, which eliminates the moment of the component F2. The
y′ Fx moment arm of F1 becomes
x
Fx 500 60° 30° O
Fx 1000 N Fy′ 90°
sin 90 sin 30 d1 4 2 tan 40 5.68 m
90° B
30° 60° F1 y
Fy 500 F = 500 N and the moment is
Fy 866 N F2
sin 60 sin 30 (c) A
x
MO 460(5.68) 2610 N m Ans.
The required scalar components are then Helpful Hint
d1 r F
(IV) Moving the force to point C eliminates the moment of the component
Fx 1000 N Fy 866 N Ans. Obtain Fx and Fy graphically and
F1. The moment arm of F2 becomes
compare your results with the calcu-
lated values. C
d2 2 4 cot 40 6.77 m F1
O d2

SAMPLE PROBLEM 2/4 a


and the moment is F2
Forces F1 and F2 act on the bracket as shown. Determine the projection Fb F1 = 100 N MO 386(6.77) 2610 N m Ans.
of their resultant R onto the b-axis. Helpful Hints
C
30° (V) By the vector expression for a moment, and by using the coordinate
The required geometry here and in
20° system indicated on the figure together with the procedures for evaluating cross
similar problems should not cause dif-
Solution. The parallelogram addition of F1 and F2 is shown in the figure. b products, we have
F2 = 80 N ficulty if the sketch is carefully drawn.
Using the law of cosines gives us This procedure is frequently the
MO r F (2i 4j) 600(i cos 40 j sin 40 ) shortest approach.
R2 (80) 2 (100)2 2(80)(100) cos 130 R 163.4 N a
F1 2610k N m The fact that points B and C are not
The figure also shows the orthogonal projection Fb of R onto the b-axis. Its 0N on the body proper should not cause
10 R The minus sign indicates that the vector is in the negative z-direction. The mag- concern, as the mathematical calcula-
length is
C 50° nitude of the vector expression is tion of the moment of a force does not
Fb 80 100 cos 50 144.3 N Ans. 80 N require that the force be on the body.
F2 50° MO 2610 N m Ans.
Note that the components of a vector are in general not equal to the projec- Alternative choices for the position
tions of the vector onto the same axes. If the a-axis had been perpendicular to Fb vector r are r d1 j 5.68j m and
the b-axis, then the projections and components of R would have been equal. r d2i 6.77i m.
b
42 Chapter 2 Force Systems Article 3/2 System Isolation and the Free-Body Diagram 115

SAMPLE PROB LEM 2/6 The free-body method is extremely important in mechanics because
T B
it ensures an accurate definition of a mechanical system and focuses at-
The trap door OA is raised by the cable AB, which passes over the small fric-
0.3 m tention on the exact meaning and application of the force laws of statics
tionless guide pulleys at B. The tension everywhere in the cable is T, and this ten-
0.4 m A and dynamics. Review the foregoing four steps for constructing a free-
sion applied at A causes a moment MO about the hinge at O. Plot the quantity MO/T
as a function of the door elevation angle over the range 0 90 and note min-
0.5 m body diagram while studying the sample free-body diagrams shown in
O Fig. 3/2 and the Sample Problems which appear at the end of the next
imum and maximum values. What is the physical significance of this ratio? θ
article.
Solution. We begin by constructing a figure which shows the tension force T
acting directly on the door, which is shown in an arbitrary angular position . It y Examples of Free-Body Diagrams
should be clear that the direction of T will vary as varies. In order to deal with
B Figure 3/2 gives four examples of mechanisms and structures to-
this variation, we write a unit vector nAB which “aims” T: rAB gether with their correct free-body diagrams. Dimensions and magni-
rAB rOB rOA T
nAB tudes are omitted for clarity. In each case we treat the entire system as
rAB rAB rOB A
d rOA
Using the x-y coordinates of our figure, we can write
O θ
x
rOB 0.4j m and rOA 0.5(cos i sin j) m SAMPLE FREE-BODY DIAGRAMS

So Mechanical System Free-Body Diagram of Isolated Body


Helpful Hints
1. Plane truss
rAB rOB rOA 0.4j (0.5)(cos i sin j)
Recall that any unit vector can be
0.5 cos i (0.4 0.5 sin )j m Weight of truss P
written as a vector divided by its assumed negligible
and magnitude. In this case the vector in compared with P P y
the numerator is a position vector.
rAB (0.5 cos )2 (0.4 0.5 sin )2
0.5 A B Ax x
0.41 0.4 sin m
Ay By
The desired unit vector is 0.4
2. Cantilever beam V
rAB 0.5 cos i (0.4 0.5 sin )j 0.3
nAB
MO
——– , m F3 F2 F1 F3 F2 F1
rAB
0.41 0.4 sin T 0.2

Our tension vector can now be written as 0.1


F y
0.5 cos i (0.4 0.5 sin )j 0 A Mass m
M
T TnAB T 0 10 20 30 40 50 60 70 80 90
0.41 0.4 sin W = mg
θ, deg
x
The moment of T about point O, as a vector, is MO rOB T, where rOB 0.4j m, or
3. Beam
0.5 cos i (0.4 0.5 sin )j Recall that any vector may be writ-
MO 0.4j T Smooth surface M M
0.41 0.4 sin ten as a magnitude times an “aim- contact at A.
0.2T cos ing” unit vector. Mass m N y
k A
0.41 0.4 sin In the expression M r F, the po- P P
The magnitude of MO is
B Bx W = mg x
sition vector r runs from the mo-
By
0.2T cos ment center to any point on the line
MO of action of F. Here, rOB is more con- 4. Rigid system of interconnected bodies
0.41 0.4 sin analyzed as a single unit y
venient than rOA.
and the requested ratio is P Weight of mechanism P
neglected
MO 0.2 cos x
Ans.
T 0.41 0.4 sin
which is plotted in the accompanying graph. The expression MO/T is the moment
arm d (in meters) which runs from O to the line of action of T. It has a maximum m W = mg
value of 0.4 m at 53.1 (at which point T is horizontal) and a minimum value of A B Bx
0 at 90 (at which point T is vertical). The expression is valid even if T varies. Ay By
This sample problem treats moments in two-dimensional force systems, and
it also points out the advantages of carrying out a solution for an arbitrary posi-
tion, so that behavior over a range of positions can be examined.
Figure 3/2

Article 3/3 Equilibrium Conditions 127 128 Chapter 3 Equilibrium

SAMPLE PROBLEM 3 /1 y SAMPLE PROB LEM 3 /2 T


Determine the magnitudes of the forces C and T, which, along with the Calculate the tension T in the cable which supports the 1000-lb load with θ = 30°
other three forces shown, act on the bridge-truss joint. the pulley arrangement shown. Each pulley is free to rotate about its bearing, C
C
and the weights of all parts are small compared with the load. Find the magni-
3 kN tude of the total force on the bearing of pulley C.
x′
Solution. The given sketch constitutes the free-body diagram of the isolated 20°
B
section of the joint in question and shows the five forces which are in equilibrium. y′
T
Solution. The free-body diagram of each pulley is drawn in its relative posi-
Solution 1 (scalar algebra). For the x-y axes as shown we have tion to the others. We begin with pulley A, which includes the only known force. A
40°
16 kN 8 kN With the unspecified pulley radius designated by r, the equilibrium of moments
x 1000 lb
[ΣFx 0] 8 T cos 40 C sin 20 16 0 about its center O and the equilibrium of forces in the vertical direction require

0.766T 0.342C 8 (a) [ΣMO 0] T1r T2 r 0 T1 T2 T


30°
[ΣFy 0] T sin 40 C cos 20 3 0 Helpful Hints [ΣFy 0] T1 T2 1000 0 2T1 1000 T1 T2 500 lb C
Fx
0.643T 0.940C 3 (b) Since this is a problem of concur- From the example of pulley A we may write the equilibrium of forces on pulley B
rent forces, no moment equation is by inspection as T3
Simultaneous solution of Eqs. (a) and (b) produces Fy T4
necessary.
T3 T4 T2/2 250 lb
T 9.09 kN C 3.03 kN Ans. B y
The selection of reference axes to fa-
For pulley C the angle 30 in no way affects the moment of T about the cen-
cilitate computation is always an im-
Solution II (scalar algebra). To avoid a simultaneous solution, we may use axes ter of the pulley, so that moment equilibrium requires T2
portant consideration. Alternatively T1
x -y with the first summation in the y -direction to eliminate reference to T. Thus, T T3 or T 250 lb Ans. x
in this example we could take a set O A
of axes along and normal to the di-
[ΣFy 0] C cos 20 3 cos 40 8 sin 40 16 sin 40 0 rection of C and employ a force sum- Equilibrium of the pulley in the x- and y-directions requires
mation normal to C to eliminate it. 1000 lb
C 3.03 kN Ans.
[ΣFx 0] 250 cos 30 Fx 0 Fx 217 lb
[ΣFx 0] T 8 cos 40 16 cos 40 3 sin 40 3.03 sin 20 0 Helpful Hint
[ΣFy 0] Fy 250 sin 30 250 0 Fy 125 lb
T 9.09 kN Ans. Clearly the radius r does not influence
[F Fx2 Fy 2] F (217)2 (125)2 250 lb Ans. the results. Once we have analyzed a
simple pulley, the results should be
Solution III (vector algebra). With unit vectors i and j in the x- and y-direc- perfectly clear by inspection.
tions, the zero summation of forces for equilibrium yields the vector equation SAMPLE PROB LEM 3 /3
[ΣF 0] 8i (T cos 40 )i (T sin 40 )j 3j (C sin 20 )i The uniform 100-kg I-beam is supported initially by its end rollers on the
horizontal surface at A and B. By means of the cable at C it is desired to elevate P
(C cos 20 )j 16i 0
end B to a position 3 m above end A. Determine the required tension P, the reac-
tion at A, and the angle made by the beam with the horizontal in the elevated
Equating the coefficients of the i- and j-terms to zero gives
position.
6m C 2m
8 T cos 40 C sin 20 16 0 A B

T sin 40 3 C cos 20 0
Solution. In constructing the free-body diagram, we note that the reaction on
which are the same, of course, as Eqs. (a) and (b), which we solved above. the roller at A and the weight are vertical forces. Consequently, in the absence of P
P B
other horizontal forces, P must also be vertical. From Sample Problem 3/2 we 2m
20° y
C see immediately that the tension P in the cable equals the tension P applied to 2m C
Solution IV (geometric). The polygon representing the zero vector sum of T 8 kN
the beam at C. 3m
the five forces is shown. Equations (a) and (b) are seen immediately to give the
3 kN 4m
Moment equilibrium about A eliminates force R and gives
projections of the vectors onto the x- and y-directions. Similarly, projections onto 40° θ 100(9.81) N
A x
the x - and y -directions give the alternative equations in Solution II. 16 kN [ΣMA 0] P(6 cos ) 981(4 cos ) 0 P 654 N Ans.
A graphical solution is easily obtained. The known vectors are laid off head-
Equilibrium of vertical forces requires
to-tail to some convenient scale, and the directions of T and C are then drawn to R
close the polygon. The resulting intersection at point P completes the solution, [ΣFy 0] 654 R 981 0 R 327 N Ans.
thus enabling us to measure the magnitudes of T and C directly from the draw- The known vectors may be added in
Helpful Hint
any order desired, but they must be The angle depends only on the specified geometry and is
ing to whatever degree of accuracy we incorporate in the construction.
added before the unknown vectors. sin 3/8 22.0 Ans. Clearly the equilibrium of this paral-
lel force system is independent of .
Article 3/3 Equilibrium Conditions 129 180 Chapter 4 Structures

SAMPLE PROBLEM 3 /4 SAMPLE PROB LEM 4 /1


Determine the magnitude T of the tension in the supporting cable and the Compute the force in each member of the loaded cantilever truss by the
magnitude of the force on the pin at A for the jib crane shown. The beam AB is a method of joints. B 5m D
standard 0.5-m I-beam with a mass of 95 kg per meter of length.
0.25 m
A 25° B 5m 5m 5m 5m
0.5 m Solution. If it were not desired to calculate the external reactions at D and E,
Algebraic solution. The system is symmetrical about the vertical x-y plane the analysis for a cantilever truss could begin with the joint at the loaded end.
through the center of the beam, so the problem may be analyzed as the equilib- However, this truss will be analyzed completely, so the first step will be to com- A
5m C 5m E
0.12 m 1.5 m
rium of a coplanar force system. The free-body diagram of the beam is shown in pute the external forces at D and E from the free-body diagram of the truss as a
the figure with the pin reaction at A represented in terms of its two rectangular whole. The equations of equilibrium give
10 kN 30 kN 20 kN
components. The weight of the beam is 95(10 3)(5)9.81 4.66 kN and acts
through its center. Note that there are three unknowns Ax, Ay, and T, which may 5m [ΣME 0] 5T 20(5) 30(10) 0 T 80 kN T
be found from the three equations of equilibrium. We begin with a moment
[ΣFx 0] 80 cos 30 Ex 0 Ex 69.3 kN 5m 30°
equation about A, which eliminates two of the three unknowns from the equa- y T
y 60°
tion. In applying the moment equation about A, it is simpler to consider the mo- [ΣFy 0] 80 sin 30 Ey 20 30 0 Ey 10 kN
ments of the x- and y-components of T than it is to compute the perpendicular Ax 25°
5m
distance from T to A. Hence, with the counterclockwise sense as positive we x
Next we draw free-body diagrams showing the forces acting on each of the
write x
Ay connecting pins. The correctness of the assigned directions of the forces is veri- Ex
4.66 kN fied when each joint is considered in sequence. There should be no question 5m 5m
[ΣMA 0] (T cos 25 )0.25 (T sin 25 )(5 0.12) 10 kN about the correct direction of the forces on joint A. Equilibrium requires
10(5 1.5 0.12) 4.66(2.5 0.12) 0 Free-body diagram 30 kN 20 kN Ey
[ΣFy 0] 0.866AB 30 0 AB 34.6 kN T Ans.
from which T 19.61 kN Ans. Helpful Hints
The justification for this step is [ΣFx 0] AC 0.5(34.6) 0 AC 17.32 kN C Ans.
Equating the sums of forces in the x- and y-directions to zero gives y
Varignon’s theorem, explained in where T stands for tension and C stands for compression. AB
[ΣFx 0] Ax 19.61 cos 25 0 Ax 17.77 kN Art. 2/4. Be prepared to take full ad- Joint B must be analyzed next, since there are more than two unknown
vantage of this principle frequently. forces on joint C. The force BC must provide an upward component, in which
[ΣFy 0] Ay 19.61 sin 25 4.66 10 0 Ay 6.37 kN 60° BD
The calculation of moments in two- case BD must balance the force to the left. Again the forces are obtained from AC x
AB = 60°
[A Ax 2 Ay 2] A (17.77)2 (6.37)2 18.88 kN Ans. dimensional problems is generally 34.6 kN 60°
handled more simply by scalar alge- [ΣFy 0] 0.866BC 0.866(34.6) 0 BC 34.6 kN C Ans. BC
bra than by the vector cross product 30 kN
r F. In three dimensions, as we will [ΣFx 0] BD 2(0.5)(34.6) 0 BD 34.6 kN T Ans.
see later, the reverse is often the case. Joint A Joint B
Graphical solution. The principle that three forces in equilibrium must be Joint C now contains only two unknowns, and these are found in the same
concurrent is utilized for a graphical solution by combining the two known verti- The direction of the force at A could way as before:
cal forces of 4.66 and 10 kN into a single 14.66-kN force, located as shown on the Helpful Hint
be easily calculated if desired. How-
modified free-body diagram of the beam in the lower figure. The position of this ever, in designing the pin A or in [ΣFy 0] 0.866CD 0.866(34.6) 20 0 It should be stressed that the ten-
resultant load may easily be determined graphically or algebraically. The inter- checking its strength, it is only the sion/compression designation refers
section of the 14.66-kN force with the line of action of the unknown tension T magnitude of the force that matters. CD 57.7 kN T Ans. to the member, not the joint. Note
defines the point of concurrency O through which the pin reaction A must pass. that we draw the force arrow on the
O [ΣFx 0] CE 17.32 0.5(34.6) 0.5(57.7) 0
The unknown magnitudes of T and A may now be found by adding the forces same side of the joint as the member
head-to-tail to form the closed equilibrium polygon of forces, thus satisfying T CE 63.5 kN C Ans. which exerts the force. In this way
Ax 25°
their zero vector sum. After the known vertical load is laid off to a convenient tension (arrow away from the joint)
scale, as shown in the lower part of the figure, a line representing the given di- A Finally, from joint E there results is distinguished from compression
Ay
rection of the tension T is drawn through the tip of the 14.66-kN vector. Like- 4.66 kN (arrow toward the joint).
wise a line representing the direction of the pin reaction A, determined from the 10 kN [ΣFy 0] 0.866DE 10 DE 11.55 kN C Ans.
concurrency established with the free-body diagram, is drawn through the tail of BC =
14.66 kN and the equation ΣFx 0 checks. 34.6 kN
the 14.66-kN vector. The intersection of the lines representing vectors T and A Note that the weights of the truss members have been neglected in compari-
kN CD DE
establishes the magnitudes T and A necessary to make the vector sum of the .88
18 son with the external loads.
forces equal to zero. These magnitudes are scaled from the diagram. The x- and A= 60° 60° 60° 69.3 kN
y-components of A may be constructed on the force polygon if desired. 14.66 kN
AC = CE CE =
17.32 kN 63.5 kN
T=1
9.61 20 kN 10 kN
k N
Graphical solution Joint C Joint E

Article 4/3 Method of Joints 181 Article 4/4 Method of Sections 191

SAMPLE PROBLEM 4 /2 SAMPLE PROB LEM 4 /4


A 10 kN 1 2
B D
The simple truss shown supports the two loads, each of magnitude L. Deter- Calculate the force in member DJ of the Howe roof truss illustrated. Ne- 10 kN
mine the forces in members DE, DF, DG, and CD. glect any horizontal components of force at the supports. C E
C
B F 6m
2R
A G
Solution. First of all, we note that the curved members of this simple truss H D L K J I H
Solution. It is not possible to pass a section through DJ without cutting four
are all two-force members, so that the effect of each curved member within the R αα G 10 kN
members whose forces are unknown. Although three of these cut by section 2 are
truss is the same as that of a straight member. α 6 panels at 4 m
α concurrent at J and therefore the moment equation about J could be used to ob-
We can begin with joint E because there are only two unknown member E
O F tain DE, the force in DJ cannot be obtained from the remaining two equilibrium
forces acting there. With reference to the free-body diagram and accompanying 10 kN
principles. It is necessary to consider first the adjacent section 1 before analyzing
geometry for joint E, we note that 180 11.25 90 78.8 .
α = 22.5° section 2. 10 kN
L L C CD Section 1
The free-body diagram for section 1 is drawn and includes the reaction of
[ΣFy 0] DE sin 78.8 L 0 DE 1.020L T Ans.
18.33 kN at A, which is previously calculated from the equilibrium of the truss
DE CJ
[ΣFx 0] EF DE cos 78.8 0 EF 0.1989L C 11.25° as a whole. In assigning the proper directions for the forces acting on the three
cut members, we see that a balance of moments about A eliminates the effects of A J
y JK
We must now move to joint F, as there are still three unknown members at joint D CD and JK and clearly requires that CJ be up and to the left. A balance of mo-
D. From the geometric diagram, 11.25°
β ments about C eliminates the effect of the three forces concurrent at C and indi- 18.33 kN
x β cates that JK must be to the right to supply sufficient counterclockwise moment.
2R sin 22.5 EF O
tan 1
42.1 11.25°
E Again it should be fairly obvious that the lower chord is under tension because of Helpful Hints
2R cos 22.5 R the bending tendency of the truss. Although it should also be apparent that the
OD = OE = 2R There is no harm in assigning one or
top chord is under compression, for purposes of illustration the force in CD will
From the free-body diagram of joint F, L more of the forces in the wrong di-
be arbitrarily assigned as tension.
Joint E rection, as long as the calculations
By the analysis of section 1, CJ is obtained from
[ΣFx 0] GF cos 67.5 DF cos 42.1 0.1989L 0 are consistent with the assumption.
[ΣMA 0] 0.707CJ(12) 10(4) 10(8) 0 CJ 14.14 kN C A negative answer will show the
[ΣFy 0] GF sin 67.5 DF sin 42.1 L 0
Helpful Hint need for reversing the direction of
Simultaneous solution of these two equations yields In this equation the moment of CJ is calculated by considering its horizontal and the force.
Rather than calculate and use the vertical components acting at point J. Equilibrium of moments about J requires
angle 78.8 in the force equa- If desired, the direction of CD may
GF 0.646L T DF 0.601L T Ans.
tions, we could have used the 11.25 [ΣMJ 0] 0.894CD(6) 18.33(12) 10(4) 10(8) 0 be changed on the free-body diagram
angle directly. and the algebraic sign of CD re-
For member DG, we move to the free-body diagram of joint D and the accompa-
CD 18.63 kN versed in the calculations, or else the
nying geometry.
work may be left as it stands with a
The moment of CD about J is calculated here by considering its two components
GF note stating the proper direction.
1 2R cos 22.5 2R cos 45 as acting through D. The minus sign indicates that CD was assigned in the
tan 33.8 DF
2R sin 45 2R sin 22.5 wrong direction.
D 10 kN
2R 10 kN
67.5° γ 22.5° Hence, CD 18.63 kN C DE
1 2R sin 22.5 R sin 45 γ
tan 2.92 Section 2
2R cos 22.5 R cos 45 0.1989L O From the free-body diagram of section 2, which now includes the known
F E DJ
value of CJ, a balance of moments about G is seen to eliminate DE and JK. 14.14 kN
Then from joint D: OF = FE = R Thus, A G
L JK J
[ΣFx 0] DG cos 2.92 CD sin 33.8 0.601L sin 47.9 1.020L cos 78.8 0 Joint F
[ΣMG 0] 12DJ 10(16) 10(20) 18.33(24) 14.14(0.707)(12) 0 18.33 kN
[ΣFy 0] DG sin 2.92 CD cos 33.8 0.601L cos 47.9 1.020L sin 78.8 0 DJ 16.67 kN T Ans.
The simultaneous solution is Observe that a section through mem-
CD Again the moment of CJ is determined from its components considered to be act- bers CD, DJ, and DE could be taken
C
ing at J. The answer for DJ is positive, so that the assumed tensile direction is which would cut only three unknown
CD 1.617L T DG 1.147L or DG 1.147L C Ans. δ correct.
δ members. However, since the forces
hor. An alternative approach to the entire problem is to utilize section 1 to deter-
Note that is shown exaggerated in the accompanying figures. D in these three members are all con-
G mine CD and then use the method of joints applied at D to determine DJ. current at D, a moment equation
β 22.5°
DG 2R
about D would yield no information
47.9° O
22.5° about them. The remaining two force
DF = 0.601L DE = 1.020L equations would not be sufficient to
OG = CG = R
Joint D solve for the three unknowns.
206 Chapter 4 Structures Article 4/6 Frames and Machines 207

SAMPLE PROB LEM 4 /6 SAMPLE PROB LEM 4 /7 B


3m 2m 30 lb
The frame supports the 400-kg load in the manner shown. Neglect the A Neglect the weight of the frame and compute the forces acting on all of its
weights of the members compared with the forces induced by the load and com- 1.5 m members. 20″
pute the horizontal and vertical components of all forces acting on each of the 12″
0.5 m R
members. 0.5 m F
B D
1.5 m E 50 lb
F Solution. We note first that the frame is not a rigid unit when removed from
C 20″ 12″
its supports since BDEF is a movable quadrilateral and not a rigid triangle. Con- E
Solution. We observe first that the three supporting members which consti- 1.5 m sequently the external reactions cannot be completely determined until the indi-
A C
tute the frame form a rigid assembly that can be analyzed as a single unit. We vidual members are analyzed. However, we can determine the vertical
also observe that the arrangement of the external supports makes the frame sta- D 400 kg components of the reactions at A and C from the free-body diagram of the frame 30″
tically determinate. as a whole. Thus,
From the free-body diagram of the entire frame we determine the external 30 lb
reactions. Thus, Helpful Hints [ΣMC 0] 50(12) 30(40) 30Ay 0 Ay 60 lb Ans. 3
4
[Σ M A 0] 5.5(0.4)(9.81) 5D 0 D 4.32 kN We see that the frame corresponds to [ΣFy 0] Cy 50(4/5) 60 0 Cy 100 lb Ans.
the category illustrated in Fig. 4/14a.
[ΣFx 0] Ax 4.32 0 Ax 4.32 kN Without this observation, the prob- Next we dismember the frame and draw the free-body diagram of each part.
50 lb
lem solution would be much longer, Since EF is a two-force member, the direction of the force at E on ED and at F on
[ΣFy 0] Ay 3.92 0 Ay 3.92 kN y
because the three equilibrium equa- AB is known. We assume that the 30-lb force is applied to the pin as a part of
tions for member BF would contain member BC. There should be no difficulty in assigning the correct directions for x
Next we dismember the frame and draw a separate free-body diagram of Ax
four unknowns: Bx, By, Ex, and Ey. forces E, F, D, and Bx. The direction of By, however, may not be assigned by inspec- Cx
each member. The diagrams are arranged in their approximate relative positions tion and therefore is arbitrarily shown as downward on AB and upward on BC.
Note that the direction of the line
to aid in keeping track of the common forces of interaction. The external reac-
joining the two points of force appli- Ay Cy
tions just obtained are entered onto the diagram for AD. Other known forces are
cation, and not the shape of the mem-
the 3.92-kN forces exerted by the shaft of the pulley on the member BF, as ob-
tained from the free-body diagram of the pulley. The cable tension of 3.92 kN is
ber, determines the direction of the Member ED. The two unknowns are easily obtained by
Helpful Hints
forces acting on a two-force member.
also shown acting on AD at its attachment point.
[ΣMD 0] 50(12) 12E 0 E 50 lb Ans. We see that this frame corresponds to
Next, the components of all unknown forces are shown on the diagrams.
Ay the category illustrated in Fig. 4/14b.
Here we observe that CE is a two-force member. The force components on CE [ΣF 0] D 50 50 0 D 100 lb Ans.
have equal and opposite reactions, which are shown on BF at E and on AD at C. y The directions of Ax and Cx are not
We may not recognize the actual sense of the components at B at first glance, so Ax obvious initially and can be assigned
they may be arbitrarily but consistently assigned. arbitrarily, to be corrected later if
The solution may proceed by use of a moment equation about B or E for x necessary.
Member EF. Clearly F is equal and opposite to E with the magnitude of 50 lb. Alternatively the 30-lb force could
member BF, followed by the two force equations. Thus,
be applied to the pin considered a
1 part of BA, with a resulting change
[ΣMB 0] 3.92(5) 2 Ex(3) 0 Ex 13.08 kN Ans. 0.4(9.81) Member AB. Since F is now known, we solve for Bx, Ax, and By from in the reaction Bx.
= 3.92 kN
[ΣFy 0] By 3.92 13.08/2 0 By 2.62 kN Ans.
[ΣMA 0] 50(3/5)(20) Bx(40) 0 Bx 15 lb Ans. By By
[ΣFx 0] Bx 3.92 13.08 0 Bx 9.15 kN Ans. D
[ΣFx 0] Ax 15 50(3/5) 0 Ax 15 lb Ans. Bx Bx 30 lb
Positive numerical values of the unknowns mean that we assumed their direc- y 3
tions correctly on the free-body diagrams. The value of Cx Ex 13.08 kN ob- [ΣFy 0] 50(4/5) 60 By 0 By 20 lb Ans. 4
3.92 kN F
tained by inspection of the free-body diagram of CE is now entered onto the Ay = 3.92 kN x
The minus sign shows that we assigned By in the wrong direction. 3
diagram for AD, along with the values of Bx and By just determined. The equa- 3.92 kN
4 D
tions of equilibrium may now be applied to member AD as a check, since all the 3.92 kN
Ax = 3.92 kN D
forces acting on it have already been computed. The equations give 4.32 kN 3.92 kN F
3.92 kN Member BC. The results for Bx, By, and D are now transferred to BC, and the
Ex E
By remaining unknown Cx is found from
[Σ M C 0] 4.32(3.5) 4.32(1.5) 3.92(2) 9.15(1.5) 0 3 50 lb
Bx Ax Cx
1 1
– Ex 3.92 kN 4
[ΣFx 0] 4.32 13.08 9.15 3.92 4.32 0 – Cx By 2 [ΣFx 0] 30 100(3/5) 15 Cx 0 Cx 75 lb Ans.
2 Cx 2
Ex We may apply the remaining two equilibrium equations as a check. Thus, A y = 60 lb E Cy = 100 lb
[ΣFy 0] 13.08/2 2.62 3.92 0 1
Cx
[ΣFy 0] 100 ( 20) 100(4/5) 0 Alternatively we could have re-
D= 1
– Cx
4.32 kN 2 [ΣMC 0] (30 15)(40) ( 20)(30) 0 turned to the free-body diagram of
the frame as a whole and found Cx.

208 Chapter 4 Structures Article 4/6 Frames and Machines 209

SAMPLE PROB LEM 4 /8 120 36 60


SAMPLE PROB LEM 4 /9 0.2 m 0.4 m 0.6 m
0.9 m 1.4 m
The machine shown is designed as an overload protection device which re- B In the particular position shown, the excavator applies a 2.1 m
0.1 m
leases the load when it exceeds a predetermined value T. A soft metal shear pin E 24 20-kN force parallel to the ground. There are two hydraulic D
50 0.4 m
S is inserted in a hole in the lower half and is acted on by the upper half. When cylinders AC to control the arm OAB and a single cylinder E A
the total force on the pin exceeds its strength, it will break. The two halves then S DE to control arm EBIF. (a) Determine the force in the hy- 0.28 m
T A T 0.55 m O
rotate about A under the action of the tensions in BD and CD, as shown in the D draulic cylinders AC and the pressure pAC against their pis- 0.45 m
F G B C
second sketch, and rollers E and F release the eye bolt. Determine the maximum Dimensions C tons, which have an effective diameter of 95 mm. (b) Also
allowable tension T if the pin S will shear when the total force on it is 800 N. in millimeters determine the force in hydraulic cylinder DE and the pres-
Also compute the corresponding force on the hinge pin A. sure pDE against its 105-mm-diameter piston. Neglect the 3.5 m
weights of the members compared with the effects of the 20- H I
kN force. J F

Solution. Because of symmetry we analyze only one of the two hinged mem- 20 kN
bers. The upper part is chosen, and its free-body diagram along with that for the
connection at D is drawn. Because of symmetry the forces at S and A have no Released
position
x-components. The two-force members BD and CD exert forces of equal magni-
tude B C on the connection at D. Equilibrium of the connection gives Solution. (a) We begin by constructing a free-body diagram of the entire arm
y assembly. Note that we include only the dimensions necessary for this portion of
[ΣFx 0] B cos C cos T 0 2B cos T B the problem—details of the cylinders DE and GH are unnecessary at this time.
B
A T
B T/(2 cos ) θ x –– [ΣMO 0] 20 000(3.95) 2FAC cos 41.3 (0.68) 2FAC sin 41.3 (2) 0
θ 2
T
From the free-body diagram of the upper part we express the equilibrium of C S FAC 48 800 N or 48.8 kN Ans.
moments about point A. Substituting S 800 N and the expression for B gives A Helpful Hint
FAC 48 800
From FAC pAC AAC, pAC 6.89(106) Pa or 6.89 MPa Ans.
AAC 0.0952 Recall that force (pressure)(area).
T T T Helpful Hints
[Σ M A 0] (cos )(50) (sin )(36) 36(800) (26) 0 4
2 cos 2 cos 2
It is always useful to recognize sym- (b) For cylinder DF, we “cut” the assembly at a location which makes the desired
Substituting sin /cos tan 5/12 and solving for T give metry. Here it tells us that the forces cylinder force external to our free-body diagram. This means isolating the verti-
acting on the two parts behave as cal arm EBIF along with the bucket and its applied force.
5(36) mirror images of each other with re-
T 25 13 28 800 spect to the x-axis. Thus, we cannot [ΣMB 0] 20 000(3.5) FDE cos 11.31 (0.73) FDE sin 11.31 (0.4) 0
2(12)
have an action on one member in the
T 1477 N or T 1.477 kN Ans. plus x-direction and its reaction on FDE 88 100 N or 88.1 kN Ans.
the other member in the negative FDE 88 100
Finally, equilibrium in the y-direction gives us x-direction. Consequently the forces p 10.18(106) Pa or 10.18 MPa Ans.
DE ADE 0.1052
at S and A have no x-components.
[ΣFy 0] S B sin A 0 4
Be careful not to forget the moment of
1477 5
800 A 0 A 492 N Ans. the y-component of B. Note that our
2(12/13) 13
units here are newton-millimeters.
0.4 m
2m
β
D
A Oy E
0.68 m FDE
2FAC By
0.73 m
α O Ox
C B Bx

3.95 m
3.5 m
0.1 + 0.4
(
β = tan−1 –––––––– = 11.31°
0.4 + 2.1 )
(
0.4 + 0.28 + 0.55
α = tan−1 –––––––––––––– = 41.3°
1.4 )
20 kN 20 kN

(a) (b)
256 Chapter 5 Distributed Forces 242 Chapter 5 Distributed Forces

SAMPLE PROB LEM 5/6 y SAMPLE PROB LEM 5 /1


Locate the centroid of the shaded area. 12″ Centroid of a circular arc. Locate the centroid of a circular arc as shown in
the figure. r
4″
Solution. The composite area is divided into the four elementary shapes α C
shown in the lower figure. The centroid locations of all these shapes may be ob- 4″ Solution. Choosing the axis of symmetry as the x-axis makes y 0. A differ- α
tained from Table D/3. Note that the areas of the “holes” (parts 3 and 4) are 3″ ential element of arc has the length dL r d expressed in polar coordinates,
taken as negative in the following table: 2″ and the x-coordinate of the element is r cos .
x
5″ Applying the first of Eqs. 5/4 and substituting L 2 r give
A x y xA yA 3″ 2″ 2″
PART in.2 in. in. in.3 in.3 [Lx x dL] (2 r)x (r cos ) r d y
r cos θ
1 120 6 5 720 600
2 rx 2r2 sin dθ
2 30 14 10/3 420 100 1 r dθ
3 14.14 6 1.273 84.8 18 4
2 r sin
4 8 12 4 96 32 x Ans.
α θ
TOTALS 127.9 959 650 For a semicircular arc 2 , which gives x 2r/ . By symmetry we see x
immediately that this result also applies to the quarter-circular arc when the α
3 measurement is made as shown.
The area counterparts to Eqs. 5/7 are now applied and yield
r
Helpful Hint
ΣAx 959
X X 7.50 in. Ans.
ΣA 127.9 It should be perfectly evident that polar coordinates are preferable to rectan-
gular coordinates to express the length of a circular arc. C C
ΣAy 650 2r/π
Y Y 5.08 in. Ans.
ΣA 127.9
r r

SAMPLE PROB LEM 5/7 6


SAMPLE PROB LEM 5 /2 y
Approximate the x-coordinate of the volume centroid of a body whose length 5
is 1 m and whose cross-sectional area varies with x as shown in the figure. 4 Centroid of a triangular area. Determine the distance h from the base of a
triangle of altitude h to the centroid of its area.
A, m2

3
2 dy
Solution. The body is divided into five sections. For each section, the average h
1
area, volume, and centroid location are determined and entered in the following Solution. The x-axis is taken to coincide with the base. A differential strip of x
0
table: 0 0.2 0.4 0.6 0.8 1.0 area dA x dy is chosen. By similar triangles x/(h y) b/h. Applying the sec- y
x, m ond of Eqs. 5/5a gives
x
Aav Volume V x Vx b
h
INTERVAL m2 m3 m m4 bh b(h y) bh2
[Ay yc dA] y y dy
2 0 h 6
0–0.2 3 0.6 0.1 0.060
0.2–0.4 4.5 0.90 0.3 0.270 h Helpful Hint
and y Ans.
0.4–0.6 5.2 1.04 0.5 0.520 3 We save one integration here by
0.6–0.8 5.2 1.04 0.7 0.728 using the first-order element of area.
This same result holds with respect to either of the other two sides of the
0.8–1.0 4.5 0.90 0.9 0.810 Recognize that dA must be expressed
triangle considered a new base with corresponding new altitude. Thus, the cen-
TOTALS 4.48 2.388 troid lies at the intersection of the medians, since the distance of this point from in terms of the integration variable
any side is one-third the altitude of the triangle with that side considered the y; hence, x ƒ( y) is required.
Helpful Hint
base.
ΣV x 2.388 Note that the shape of the body as a
X X 0.533 m Ans.
ΣV 4.48 function of y and z does not affect X.

Article 5/3 Centroids of Lines, Areas, and Volumes 243 244 Chapter 5 Distributed Forces

SAMPLE PROBLEM 5 /3 SAMPLE PROB LEM 5/4 y

Centroid of the area of a circular sector. Locate the centroid of the area r Locate the centroid of the area under the curve x ky3 from x 0 to x a. x = ky3
of a circular sector with respect to its vertex.
α C –
x
α
Solution I. A vertical element of area dA y dx is chosen as shown in the fig- b
C
Solution I. The x-axis is chosen as the axis of symmetry, and y is therefore ure. The x-coordinate of the centroid is found from the first of Eqs. 5/5a. Thus,

y
automatically zero. We may cover the area by moving an element in the form of
a a
a partial circular ring, as shown in the figure, from the center to the outer pe-
[Ax xc dA] x y dx xy dx
riphery. The radius of the ring is r0 and its thickness is dr0, so that its area is 0 0 a x
y
dA 2r0 dr0.
1/3 3
The x-coordinate to the centroid of the element from Sample Problem 5/1 is Substituting y (x/k) and k a/b and integrating give y
r
xc r0 sin / , where r0 replaces r in the formula. Thus, the first of Eqs. 5/5a
gives dr0 3ab 3a2b 4 b
x x 7a Ans.
α r0 4 7 x = ky3
r x
2 r0 sin α
[Ax xc dA] ( r2)x (2r0 dr0) In the solution for y from the second of Eqs. 5/5a, the coordinate to the
2 0
centroid of the rectangular element is yc y/2, where y is the height of the strip
2 3 governed by the equation of the curve x ky3. Thus, the moment principle be- y
r2 x r0 sin α
3 r sin xc = ——–––
α comes y
yc = –
2 r sin a 2
x Ans. 3ab y
3 Solution I [Ay yc dA] y y dx x
4 0 2 x dx a
1/3
Substituting y b(x/a) and integrating give
Helpful Hints
Solution II. The area may also be covered by swinging a triangle of differen-
tial area about the vertex and through the total angle of the sector. This triangle, Note carefully that we must distin- 3ab 3ab2 2
y y 5b Ans.
shown in the illustration, has an area dA (r/2)(r d ), where higher-order terms guish between the variable r0 and 4 10
are neglected. From Sample Problem 5/2 the centroid of the triangular element the constant r.
y
of area is two-thirds of its altitude from its vertex, so that the x-coordinate to the Be careful not to use r0 as the cen- xc = a +x
––––
centroid of the element is xc 32 r cos . Applying the first of Eqs. 5/5a gives troidal coordinate for the element. Solution II. The horizontal element of area shown in the lower figure may be b
2
employed in place of the vertical element. The x-coordinate to the centroid of the
x = ky3
rectangular element is seen to be xc x 12 (a x) (a x)/2, which is simply
[Ax xc dA] (r2 )x (23 r cos )(12 r2 d ) dy
y the average of the coordinates a and x of the ends of the strip. Hence,
2 r cos θ b b
r2
x 2 3 xc = – a x
3 r sin 3 [Ax xc dA] x (a x) dy
2
(a x) dy

0 0 x a–x y
2 r sin
and as before x Ans. The value of y is found from
3 α
θ b b a
x
For a semicircular area 2 , which gives x 4r/3 . By symmetry we see x
α [Ay yc dA] y (a x) dy y(a x) dy
immediately that this result also applies to the quarter-circular area where the 0 0
measurement is made as shown. Helpful Hint
It should be noted that, if we had chosen a second-order element r0 dr0 d , r where yc y for the horizontal strip. The evaluation of these integrals will check
one integration with respect to would yield the ring with which Solution I the previous results for x and y. Note that xc x for the vertical
began. On the other hand, integration with respect to r0 initially would give the element.
triangular element with which Solution II began.
Solution II

4r/3π
C
C

r r
274 Chapter 5 Distributed Forces 284 Chapter 5 Distributed Forces

SAMPLE PROB LEM 5/11 SAMPLE PROB LEM 5/14


4′ 6′
w0
Determine the equivalent concentrated load(s) and external reactions for 280 lb/ft The cantilever beam is subjected to the load intensity (force per unit length) w
the simply supported beam which is subjected to the distributed load shown. 120 lb/ft which varies as w w0 sin ( x/l). Determine the shear force V and bending mo-
A B ment M as functions of the ratio x/l.

x
Solution. The area associated with the load distribution is divided into the Helpful Hint l
rectangular and triangular areas shown. The concentrated-load values are deter- Solution. The free-body diagram of the entire beam is drawn first so that the
Note that it is usually unnecessary
mined by computing the areas, and these loads are located at the centroids of the shear force V0 and bending moment M0 which act at the supported end at x 0 R
to reduce a given distributed load to y
respective areas. can be computed. By convention V0 and M0 are shown in their positive mathe-
a single concentrated load.
Once the concentrated loads are determined, they are placed on the free- matical senses. A summation of vertical forces for equilibrium gives
w
body diagram of the beam along with the external reactions at A and B. Using M0
principles of equilibrium, we have 1
– (160) (6) = 480 lb
l l
x 2w0l
2 [ΣFy 0] V0 w dx 0 V0 w 0 sin dx x
8′ 0 0 l
5′ 160 lb/ft
[ΣMA 0] 1200(5) 480(8) RB(10) 0 x dx
120 lb/ft 120 lb/ft A summation of moments about the left end at x 0 for equilibrium gives
RB 984 lb Ans. A B V0
l l
(120) (10) = 1200 lb x
[ΣMB 0] RA(10) 1200(5) 480(2) 0 [ΣM 0] M0 x(w dx) 0 M0 w0 x sin dx
0 0 l
1200 lb 480 lb M0 M
RA 696 lb Ans. w0l2
l
w0l2
5′ 3′ x x x
M0 sin cos
A B 2 l l l 0
x
RA RB
From a free-body diagram of an arbitrary section of length x, integration of
Eq. 5/10 permits us to find the shear force internal to the beam. Thus, V0 V
V x
x 0.637
[dV w dx] dV w0 sin dx
V0 0 l
SAMPLE PROB LEM 5/12 w0l
x
2w0l w0l V
w(x) w = w0 + kx3 x x ——
V V0 cos V cos 1 w0 l
Determine the reaction at the support A of the loaded cantilever beam. 1000 N/m 2024 l 0
l
N/m
x or in dimensionless form 0
B 0 0.2 x/l 0.6 0.8 1.0
8m 0
Solution. The constants in the load distribution are found to be w0 1000 V 1 x
A 1 cos Ans. M
N/m and k 2 N/m4. The load R is then w0l l
——–
w0 l2
8 8
x4 Helpful Hints The bending moment is obtained by integration of Eq. 5/11, which gives –0.318
R w dx (1000 2x3) dx 1000x 10 050 N
0 2 0
Use caution with the units of the M x
w 0l x Helpful Hints
constants w0 and k. [dM V dx] dM 1 cos dx
The x-coordinate of the centroid of the area is found by M0 0 l
x In this case of symmetry it is clear
xw dx The student should recognize that w0l l x
1
8
M M0 x sin that the resultant R V0 2w0l/ of
x x(1000 2x3) dx the calculation of R and its location x l the load distribution acts at midspan,
R 10 050 0 0
is simply an application of centroids w0l2 w0l so that the moment requirement is
l x
1 2 5 8 as treated in Art. 5/3. M x sin 0 simply M0 Rl/2 w0l2/ . The
(500x2 l
10 050 5x ) 0 4.49 m
minus sign tells us that physically
10 050 N or in dimensionless form the bending moment at x 0 is oppo-
From the free-body diagram of the beam, we have site to that represented on the free-
4.49 m
Ax B M 1 x 1 x body diagram.
[ΣMA 0] MA (10 050)(4.49) 0 A y 1 sin Ans.
MA w0l2 l l The free-body diagram serves to
MA 45 100 N m Ans. Ay remind us that the integration
x
The variations of V/w0l and M/w0l2 with x/l are shown in the bottom figures.
limits for V as well as for x must be
[ΣFy 0] Ay 10 050 N Ans. The negative values of M/w0 l2 indicate that physically the bending moment is in
accounted for. We see that the
the direction opposite to that shown.
Note that Ax 0 by inspection. expression for V is positive, so that
the shear force is as represented on
the free-body diagram.

Article 5/7 Beams—Internal Effects 285 298 Chapter 5 Distributed Forces

SAMPLE PROBLEM 5/15 100 lb/ft SAMPLE PROB LEM 5 /16 T T


300 lb A B
Draw the shear-force and bending-moment diagrams for the loaded beam A 100-ft length of surveyor’s tape weights 0.6 lb. When the tape is stretched h
and determine the maximum moment M and its location x from the left end. between two points on the same level by a tension of 10 lb at each end, calculate
the sag h in the middle.

Solution. The support reactions are most easily obtained by considering the 4′ 4′ 2′ 2′
resultants of the distributed loads as shown on the free-body diagram of the Solution. The weight per unit length is 0.6 /100 0.006 lb /ft. The total
beam as a whole. The first interval of the beam is analyzed from the free-body di- y length is 2s 100 or s 50 ft.
200 lb 400 lb
agram of the section for 0 x 4 ft. A summation of vertical forces and a mo- 8′
–– 300 lb
ment summation about the cut section yield 3 [T 2 2 2
s T02] 102 (0.006)2 (50)2 T02
T0 9.995 lb Helpful Hint
[ΣFy 0] V 247 12.5x2
x [T T0 y] 10 9.995 0.006h
x An extra significant figure is dis-
[ΣM 0] M (12.5x2) 247x 0 M 247x 4.17x3 played here for clarity.
3 h 0.750 ft or 9.00 in. Ans.
R1 = 247 lb R2 = 653 lb
These values of V and M hold for 0 x 4 ft and are plotted for that interval in
the shear and moment diagrams shown. 12.5x2
From the free-body diagram of the section for which 4 x 8 ft, equilib- 2x x V 300 lb SAMPLE PROB LEM 5/17 300 m
–– – 100
rium in the vertical direction and a moment sum about the cut section give 3 4
12 – x The light cable supports a mass of 12 kg per meter of horizontal length and
M is suspended between the two points on the same level 300 m apart. If the sag is
[ΣFy 0] V 100(x 4) 200 247 0 V 447 100x 60 m
x 60 m, find the tension at midlength, the maximum tension, and the total length
M of the cable.
x 4 2
[ΣM 0] M 100(x 4) 200[x 3 (4)] 247x 0
2 V
247 lb 653 lb 12 kg/m
M 267 447x 50x2 100(x – 4)
Solution. With a uniform horizontal distribution of load, the solution of part y Tmax
200 lb x–4
––––
These values of V and M are plotted on the shear and moment diagrams for the (b) of Art. 5/8 applies, and we have a parabolic shape for the cable. For h 60 m,
2
interval 4 x 8 ft. L 300 m, and w 12(9.81)(10 3) kN/m the relation following Eq. 5/14 with
The analysis of the remainder of the beam is continued from the free-body M lA L/2 gives for the midlength tension
60 m
diagram of the portion of the beam to the right of a section in the next interval. x
x wL2 0.1177(300)2 T0
It should be noted that V and M are represented in their positive directions. A T0 T0 22.1 kN Ans. 75 m 75 m
vertical-force summation and a moment summation about the section yield 8h 8(60)
V –3
247 lb The maximum tension occurs at the supports and is given by Eq. 5/15b. Thus, R = 12(150)(9.81)(10 )
V 353 lb and M 2930 353x = 17.66 kN
V, lb 2
wL L
Tmax 1
These values of V and M are plotted on the shear and moment diagrams for the 2 4h Helpful Hint
247 300
interval 8 x 10 ft.
12(9.81)(10 3)(300) 300 2 Suggestion: Check the value of Tmax
The last interval may be analyzed by inspection. The shear is constant at Tmax 1 28.3 kN Ans.
0 x, ft 2 4(60) directly from the free-body diagram
300 lb, and the moment follows a straight-line relation beginning with zero at 0 4 8 10 12 of the right-hand half of the cable,
the right end of the beam. The sag-to-span ratio is 60/300 1/5 1/4. Therefore, the series expression
4.47′ from which a force polygon may be
The maximum moment occurs at x 4.47 ft, where the shear curve crosses developed in Eq. 5/16a is convergent, and we may write for the total length
drawn.
the zero axis, and the magnitude of M is obtained for this value of x by substitu- –353
tion into the expression for M for the second interval. The maximum moment is M, lb-ft 8 1 2
32 1 4
S 300 1 …
732 3 5 5 5
M 732 lb-ft Ans. …]
300[1 0.1067 0.01024
0 x, ft 329 m Ans.
As before, note that the change in moment M up to any section equals the 0 4 8 10 12
area under the shear diagram up to that section. For instance, for x 4 ft,

x
[ M V dx] M 0 (247 12.5x2) dx –600
0

3
and, as above, M 247x 4.17x
Article 5/8 Flexible Cables 299 314 Chapter 5 Distributed Forces

SAMPLE PROBLEM 5/18 300 m SAMPLE PROB LEM 5/19


A
Replace the cable of Sample Problem 5/17, which is loaded uni- y A rectangular plate, shown in vertical section AB, is 4 m high and 6 m wide 1m
formly along the horizontal, by a cable which has a mass of 12 kg per 60 m (normal to the plane of the paper) and blocks the end of a fresh-water channel 3 m
meter of its own length and supports its own weight only. The cable is deep. The plate is hinged about a horizontal axis along its upper edge through A
suspended between two points on the same level 300 m apart and has a x and is restrained from opening by the fixed ridge B which bears horizontally against
3m
sag of 60 m. Find the tension at midlength, the maximum tension, and the lower edge of the plate. Find the force B exerted on the plate by the ridge.
the total length of the cable.
B

Solution. The free-body diagram of the plate is shown in section and includes y
Solution. With a load distributed uniformly along the length of 0.33 the vertical and horizontal components of the force at A, the unspecified weight
the cable, the solution of part (c) of Art. 5/8 applies, and we have W mg of the plate, the unknown horizontal force B, and the resultant R of the Ay
a catenary shape of the cable. Equations 5/20 and 5/21 for the triangular distribution of pressure against the vertical face.
0.32 x
cable length and tension both involve the minimum tension T0 at Solution The density of fresh water is 1.000 Mg/m3 so that the average pressure is Ax
midlength, which must be found from Eq. 5/19. Thus, for x 150 m, T0 = 23.2 kN
y 60 m, and 12(9.81)(10 3) 0.1177 kN/m, we have 0.31 [pav gh] pav 1.000(9.81)(32 ) 14.72 kPa
7.06
T0 (0.1177)(150)
(——–
T0 ( The resultant R of the pressure forces against the plate becomes
2m mg 4m

60 cosh 1 0.30 R
0.1177 T0 [R pavA] R (14.72)(3)(6) 265 kN 1m
17.66
or
7.06
cosh
17.66
1 0.29 (cosh ——– – 1
T0
( This force acts through the centroid of the triangular distribution of pressure,
B
T0 T0 which is 1 m above the bottom of the plate. A zero moment summation about A
Helpful Hint
establishes the unknown force B. Thus,
This equation can be solved graphically. We compute the expres- 0.28 Note that the units of pressure gh are
sion on each side of the equals sign and plot it as a function of T0. The 22.5 23.0 23.5 24.0
[ΣMA 0] 3(265) 4B 0 B 198.7 kN Ans.
intersection of the two curves establishes the equality and determines T0, kN kg m (m) kg m 1
103 103
the correct value of T0. This plot is shown in the figure accompanying m3 s2 s2 m2
this problem and yields the solution kN/m 2
kPa.

T0 23.2 kN
SAMPLE PROB LEM 5/20
Alternatively, we may write the equation as A 25″
The air space in the closed fresh-water tank is maintained at a pressure of
Air 8″
0.80 lb/in.2 (above atmospheric). Determine the resultant force R exerted by the
17.66 7.06
ƒ(T0) cosh 1 0 air and water on the end of the tank.
T0 T0
Water 30″
and set up a computer program to calculate the value(s) of T0 which renders
ƒ(T0) 0. See Art. C/11 of Appendix C for an explanation of one applicable nu-
Solution. The pressure distribution on the end surface is shown, where p0
merical method. B
0.80 lb/in.2 The specific weight of fresh water is g 62.4/1728 0.0361 Side view End view
The maximum tension occurs for maximum y and from Eq. 5/22 is
lb/in.3 so that the increment of pressure p due to the water is
p0 A
Tmax 23.2 (0.1177)(60) 30.2 kN Ans. p h 0.0361(30) 1.083 lb/in. 2

The resultant forces R1 and R2 due to the rectangular and triangular distribu- 19″ h
From Eq. 5/20 the total length of the cable becomes R1 28″
tions of pressure, respectively, are
23.2 (0.1177)(150) R
2s 2 sinh 330 m Ans. R1 p0A1 0.80(38)(25) 760 lb
0.1177 23.2 R2
1.083
R2 pavA2 (30)(25) 406 lb
2 ∆p p0 B
Helpful Hint The resultant is then R R1 R2 760 406 1166 lb. Ans.
Note that the solution of Sample Problem 5/17 for the parabolic cable gives a Helpful Hint
We locate R by applying the moment principle about A noting that R1 acts
very close approximation to the values for the catenary even though we have through the center of the 38-in. depth and that R2 acts through the centroid of Dividing the pressure distribution
a fairly large sag. The approximation is even better for smaller sag-to-span the triangular pressure distribution 20 in. below the surface of the water and into these two parts is decidedly the
ratios. 20 8 28 in. below A. Thus, simplest way in which to make the
calculation.
[Rh ΣMA] 1166h 760(19) 406(28) h 22.1 in. Ans.

Article 5/9 Fluid Statics 315 316 Chapter 5 Distributed Forces

SAMPLE PROBLEM 5/21 O D


SAMPLE PROB LEM 5/22
Determine completely the resultant force R exerted on the cylindrical dam Determine the resultant force R exerted on the semicircular end of the
surface by the water. The density of fresh water is 1.000 Mg/m3, and the dam has water tank shown in the figure if the tank is filled to capacity. Express the result
a length b, normal to the paper, of 30 m. in terms of the radius r and the water density . r
r=4m

Solution I. We will obtain R first by a direct integration. With a horizontal


B
Solution. The circular block of water BDO is isolated and its free-body dia- strip of area dA 2x dy acted on by the pressure p gy, the increment of the
gram is drawn. The force Px is resultant force is dR p dA so that r
r
gr (1.000)(9.81)(4)
Px ghA br (30)(4) 2350 kN O D R p dA gy(2x dy) 2 g y r2 y2 dy
2 2 x 0 x
mg p y
The weight W of the water passes through the mass center G of the quarter- 2 3 x
Integrating gives R ––
circular section and is 3 gr Ans.
dy Y
G C
(4)2 The location of R is determined by using the principle of moments. Taking R
mg gV (1.000)(9.81) (30) 3700 kN Px C
4 moments about the x-axis gives
r y
Equilibrium of the section of water requires A 2 3 2 2 2
[RY y dR] 3 gr Y 2 g y r y dy
B 0
[ΣFx 0] Rx Px 2350 kN x
R Helpful Hint
y 2 3
gr4 3 r
[ΣFy 0] Ry mg 3700 kN Integrating gives 3 gr Y and Y Ans. Be very careful not to make the mis-
4 2 16
take of assuming that R passes
The resultant force R exerted by the fluid on the dam is equal and opposite
O D through the centroid of the area
to that shown acting on the fluid and is Solution II. We may use Eq. 5/25 directly to find R, where the average pres-
θ over which the pressure acts.
sure is gh and h is the coordinate to the centroid of the area over which the
[R Rx2 R y2 ] R (2350)2 (3700)2 4380 kN Ans. r r sin θ
p pressure acts. For a semicircular area, h 4r/(3 ).
The x-coordinate of the point A through which R passes may be found from the y1
principle of moments. Using B as a moment center gives R 4r r2 2 3
[R ghA] R g 3 gr Ans.
3 2
4 16 x1
2350 3700 which is the volume of the pressure-area figure.
r 4r 3 3
Px mg Ryx 0, x 2.55 m Ans. B x A The resultant R acts through the centroid C of the volume defined by the
3 3 3700
pressure-area figure. Calculation of the centroidal distance Y involves the same
Helpful Hints integral obtained in Solution I.
Alternative Solution. The force acting on the dam surface may be obtained
See note in Sample Problem 5/19
by a direct integration of the components
if there is any question about the
dRx p dA cos and dRy p dA sin units for gh. SAMPLE PROB LEM 5/23
θ
where p gh gr sin and dA b(r d ). Thus, A buoy in the form of a uniform 8-m pole 0.2 m in diameter has a mass of
This approach by integration is feasi- 200 kg and is secured at its lower end to the bottom of a fresh-water lake with
/2
/2
cos 2 1 ble here mainly because of the simple 5 m of cable. If the depth of the water is 10 m, calculate the angle made by
Rx gr2b sin cos d gr2b 2
2 gr b geometry of the circular arc.
0 4 the pole with the horizontal. 10 m
0
/2
/2
sin 2 1 5m
Ry gr2b sin2 d gr2b gr2b
0 2 4 4 Solution. The free-body diagram of the buoy shows its weight acting through
0
2 2 1 2 2
G, the vertical tension T in the anchor cable, and the buoyancy force B which
Thus, R Rx Ry 2 gr b 1 /4. Substituting the numerical values gives
passes through centroid C of the submerged portion of the buoy. Let x be the dis-
1 2 2 tance from G to the waterline. The density of fresh water is 103 kg/m3, so
R 2 (1.000)(9.81)(4 )(30) 1 /4 4380 kN Ans.
that the buoyancy force is
Since dR always passes through point O, we see that R also passes through 4m
O and, therefore, the moments of Rx and Ry about O must cancel. So we write [B gV] B 103(9.81) (0.1)2(4 x) N
Rxy1 Ryx1, which gives us G x
Moment equilibrium, ΣMA 0, about A gives
C 5m
4m
x1/y1 Rx/Ry (12 gr2b)/(14 gr2b) 2/ 4 x 200(9.81) N
200(9.81)(4 cos ) [10 3(9.81) (0.1)2(4 x)] cos 0
2
θ B
By similar triangles we see that A
1 5
Thus, x 3.14 m and sin 44.5 Ans.
x/r x1/y1 2/ and x 2r/ 2(4)/ 2.55 m Ans. 4 3.14 T
342 Chapter 6 Friction Article 6/3 Dry Friction 343

SAMPLE PROB LEM 6 /1 SAMPLE PROB LEM 6 /3


kg
Determine the maximum angle which the adjustable incline may have Determine the magnitude and direction of the friction force acting on the P 100
with the horizontal before the block of mass m begins to slip. The coefficient of m 100-kg block shown if, first, P 500 N and, second, P 100 N. The coefficient of
static friction between the block and the inclined surface is s. µs static friction is 0.20, and the coefficient of kinetic friction is 0.17. The forces are 20°
θ applied with the block initially at rest.

Solution. The free-body diagram of the block shows its weight W mg, the
normal force N, and the friction force F exerted by the incline on the block. The y Solution. There is no way of telling from the statement of the problem whether
W = mg y
friction force acts in the direction to oppose the slipping which would occur if no the block will remain in equilibrium or whether it will begin to slip following the
friction were present. application of P. It is therefore necessary that we make an assumption, so we will 100(9.81) = 981 N
Equilibrium in the x- and y-directions requires take the friction force to be up the plane, as shown by the solid arrow. From the
free-body diagram a balance of forces in both x- and y-directions gives x
x
[ΣFx 0] mg sin F 0 F mg sin F N
θ P
[ΣFx 0] P cos 20 F 981 sin 20 0
F
[ΣFy 0] mg cos N 0 N mg cos Helpful Hints
[ΣFy 0] N P sin 20 981 cos 20 0 20° F
We choose reference axes along and N
Dividing the first equation by the second gives F/N tan . Since the maximum
normal to the direction of F to
angle occurs when F Fmax sN, for impending motion we have
avoid resolving both F and N into
components. Case I. P 500 N
1
s tan max or max tan s Ans. Substitution into the first of the two equations gives
This problem describes a very simple
way to determine a static coefficient F 134.3 N
of friction. The maximum value of
is known as the angle of repose. The negative sign tells us that if the block is in equilibrium, the friction force
acting on it is in the direction opposite to that assumed and therefore is down
SAMPLE PROB LEM 6 /2 kg the plane, as represented by the dashed arrow. We cannot reach a conclusion on
10 0 the magnitude of F, however, until we verify that the surfaces are capable of
Determine the range of values which the mass m0 may have so that the 100-kg supporting 134.3 N of friction force. This may be done by substituting P 500 N
block shown in the figure will neither start moving up the plane nor slip down 20° m0
into the second equation, which gives
the plane. The coefficient of static friction for the contact surfaces is 0.30.
y N 1093 N
981 N x
Solution. The maximum value of m0 will be given by the requirement for mo- The maximum static friction force which the surfaces can support is then
T = m0 g
tion impending up the plane. The friction force on the block therefore acts down
the plane, as shown in the free-body diagram of the block for Case I in the figure. Fmax [Fmax sN] Fmax 0.20(1093) 219 N
With the weight mg 100(9.81) 981 N, the equations of equilibrium give 20°
N Since this force is greater than that required for equilibrium, we conclude that
Case I the assumption of equilibrium was correct. The answer is, then,
[ΣFy 0] N 981 cos 20 0 N 922 N
y
981 N F 134.3 N down the plane Ans.
[Fmax x
s N] Fmax 0.30(922) 277 N
T = m0 g
[ΣFx 0] m0(9.81) 277 981 sin 20 0 m0 62.4 kg Ans. Fmax
Case II. P 100 N
The minimum value of m0 is determined when motion is impending down the 20° Substitution into the two equilibrium equations gives
N
plane. The friction force on the block will act up the plane to oppose the ten- Case II F 242 N N 956 N
dency to move, as shown in the free-body diagram for Case II. Equilibrium in the
x-direction requires Helpful Hint But the maximum possible static friction force is
Helpful Hint
[ΣFx 0] m0(9.81) 277 981 sin 20 0 m0 6.01 kg Ans. We see from the results of Sample [Fmax sN] Fmax 0.20(956) 191.2 N
We should note that even though ΣFx
Problem 6/1 that the block would
It follows that 242 N of friction cannot be supported. Therefore, equilibrium cannot is no longer equal to zero, equilibrium
Thus, m0 may have any value from 6.01 to 62.4 kg, and the block will remain at slide down the incline without the re-
exist, and we obtain the correct value of the friction force by using the kinetic coeffi- does exist in the y-direction, so that
rest. straint of attachment to m0 since tan
cient of friction accompanying the motion down the plane. Hence, the answer is ΣFy 0. Therefore, the normal force
In both cases equilibrium requires that the resultant of Fmax and N be con- 20 0.30. Thus, a value of m0 will
N is 956 N whether or not the block is
current with the 981-N weight and the tension T. be required to maintain equilibrium.
[Fk kN] F 0.17(956) 162.5 N up the plane Ans. in equilibrium.

344 Chapter 6 Friction Article 6/3 Dry Friction 345

SAMPLE PROB LEM 6 /4 b SAMPLE PROB LEM 6 /5


The homogeneous rectangular block of mass m, width b, and height H is The three flat blocks are positioned on the 30 incline as shown, and a force
placed on the horizontal surface and subjected to a horizontal force P which P P parallel to the incline is applied to the middle block. The upper block is pre-
moves the block along the surface with a constant velocity. The coefficient of ki- H m vented from moving by a wire which attaches it to the fixed support. The coeffi-
kg
netic friction between the block and the surface is k. Determine (a) the greatest h cient of static friction for each of the three pairs of mating surfaces is shown. 30
value which h may have so that the block will slide without tipping over and Determine the maximum value which P may have before any slipping takes kg
0 50
(b) the location of a point C on the bottom face of the block through which the place. = 0.3 kg
µs 40
resultant of the friction and normal forces acts if h H/2. .45
P 0 =0
= 0.4 µs
y Solution. The free-body diagram of each block is drawn. The friction forces µs
30°
Solution. (a) With the block on the verge of tipping, we see that the entire re- are assigned in the directions to oppose the relative motion which would occur if
action between the plane and the block will necessarily be at A. The free-body no friction were present. There are two possible conditions for impending mo-
tion. Either the 50-kg block slips and the 40-kg block remains in place, or the 50- y 30(9.81) N
diagram of the block shows this condition. Since slipping occurs, the friction B T
P and 40-kg blocks move together with slipping occurring between the 40-kg block
force is the limiting value k N, and the angle becomes tan 1 k. The resul-
tant of Fk and N passes through a point B through which P must also pass, since θ G and the incline. 30°
h The normal forces, which are in the y-direction, may be determined without
three coplanar forces in equilibrium are concurrent. Hence, from the geometry F1
Fk mg reference to the friction forces, which are all in the x-direction. Thus,
of the block A
x
N1
b [ΣFy 0] (30- kg) N1 30(9.81) cos 30 0 N1 255 N x
b/2 b θ —
tan k h Ans. 2
h 2 k N (50- kg) N2 50(9.81) cos 30 255 0 N2 680 N
y
If h were greater than this value, moment equilibrium about A would not be (40- kg) N3 40(9.81) cos 30 680 0 N3 1019 N
satisfied, and the block would tip over. 50(9.81) N
N1
Alternatively, we may find h by combining the equilibrium requirements for We will assume arbitrarily that only the 50-kg block slips, so that the 40-kg
the x- and y-directions with the moment-equilibrium equation about A. Thus, G block remains in place. Thus, for impending slippage at both surfaces of the F1
P 50-kg block, we have
[ΣFy 0] N mg 0 N mg θ H

mg N2
2 [Fmax
C s N] F1 0.30(255) 76.5 N F2 0.40(680) 272 N P F2
[ΣFx 0] Fk P 0 P Fk kN kmg Fk x
b mgb mgb b θ The assumed equilibrium of forces at impending motion for the 50-kg block x
[Σ M A 0] Ph mg 0 h Ans. gives y
2 2P 2 kmg 2 k N
(b) With h H/2 we see from the free-body diagram for case (b) that the re- 40(9.81) N
[ΣFx 0] P 76.5 272 50(9.81) sin 30 0 P 103.1 N N2
sultant of Fk and N passes through a point C which is a distance x to the left of
the vertical centerline through G. The angle is still tan 1 k as long as Helpful Hints F2
We now check on the validity of our initial assumption. For the 40-kg block
the block is slipping. Thus, from the geometry of the figure we have with F2 272 N the friction force F3 would be given by
Recall that the equilibrium equa-
tions apply to a body moving with a N3
x [ΣFx 0] 272 40(9.81) sin 30 F3 0 F3 468 N x F3
tan k so x k H/2 Ans. constant velocity (zero acceleration)
H/2
just as well as to a body at rest.
But the maximum possible value of F3 is F3 s N3 0.45(1019) 459 N. Thus,
If we were to replace k by the static coefficient s, then our solutions would Helpful Hints
Alternatively, we could equate the 468 N cannot be supported and our initial assumption was wrong. We conclude,
describe the conditions under which the block is (a) on the verge of tipping and therefore, that slipping occurs first between the 40-kg block and the incline. In the absence of friction the middle
(b) on the verge of slipping, both from a rest position. moments about G to zero, which
would give us F(H/2) Nx 0. Thus, With the corrected value F3 459 N, equilibrium of the 40-kg block for its im- block, under the influence of P,
with Fk pending motion requires would have a greater movement
k N we get x x H/2.
than the 40-kg block, and the fric-
[ΣFx 0] F2 40(9.81) sin 30 459 0 F2 263 N tion force F2 will be in the direction
to oppose this motion as shown.
Equilibrium of the 50-kg block gives, finally,
We see now that F2 is less than
[ΣFx 0] P 50(9.81) sin 30 263 76.5 0 s N2 272 N.

P 93.8 N Ans.

Thus, with P 93.8 N, motion impends for the 50-kg and 40-kg blocks as a unit.
Article 6/5 Screws 361 360 Chapter 6 Friction

SAMPLE PROBLEM 6 /6 P W W

The horizontal position of the 500-kg rectangular block of concrete is ad- M


P = ––
r L L
justed by the 5 wedge under the action of the force P. If the coefficient of static α tan α = ––– α
2π r M
friction for both wedge surfaces is 0.30 and if the coefficient of static friction 5° P = ––
500 kg r R α
between the block and the horizontal surface is 0.60, determine the least force P
α φ R
required to move the block. φ
2π r
(a) To raise load (b) To lower load (α < φ )

Solution. The free-body diagrams of the wedge and the block are drawn with P
W
the reactions R1, R2, and R3 inclined with respect to their normals by the
amount of the friction angles for impending motion. The friction angle for limit- b
–R2 W = 500(9.81) N
ing static friction is given by tan 1 . Each of the two friction angles is com- 5° φ1 M
P = ––
puted and shown on the diagram. φ1 r
φ1 α φ
We start our vector diagram expressing the equilibrium of the block at a R2 a
R1 α
convenient point A and draw the only known vector, the weight W of the block.
Next we add R3, whose 31.0 inclination from the vertical is now known. The b R
vector R2, whose 16.70 inclination from the horizontal is also known, must φ 1 = tan–1 0.30 a R3 (c) To lower load (α > φ )
= 16.70° φ2
close the polygon for equilibrium. Thus, point B on the lower polygon is deter- φ 2 = tan–1 0.60
mined by the intersection of the known directions of R3 and R2, and their mag- = 31.0° Figure 6/7
nitudes become known.
For the wedge we draw R2, which is now known, and add R1, whose direc-
tion is known. The directions of R1 and P intersect at C, thus giving us the solu- R1 C
Conditions for Unwinding
tion for the magnitude of P. 16.70° + 5° = 21.7° If the moment M is removed, the friction force changes direction so
Wedge P
16.70°
that is measured to the other side of the normal to the thread. The
–R2 screw will remain in place and be self-locking provided that , and
R2
Algebraic solution. The simplest choice of reference axes for calculation B will be on the verge of unwinding if .
purposes is, for the block, in the direction a-a normal to R3 and, for the wedge, in 16.70° A To lower the load by unwinding the screw, we must reverse the di-
the direction b-b normal to R1. The angle between R2 and the a-direction is rection of M as long as . This condition is illustrated in Fig. 6/7b
16.70 31.0 47.7 . Thus, for the block Block for our simulated thread on the fixed incline. An equivalent force P
R3 M/r must be applied to the thread to pull it down the incline. From the
[ΣFa 0] 500(9.81) sin 31.0 R2 cos 47.7 0 31.0° W = 4905 N triangle of vectors we therefore obtain the moment required to lower
R2 3750 N the screw, which is

For the wedge the angle between R2 and the b-direction is 90 (2 1 M Wr tan ( ) (6/3a)
5) 51.6 , and the angle between P and the b-direction is 1 5 21.7 . Thus,
If , the screw will unwind by itself, and Fig. 6/7c shows that the
[ΣFb 0] 3750 cos 51.6 P cos 21.7 0 moment required to prevent unwinding is
Helpful Hints
P 2500 N Ans.
Be certain to note that the reactions M Wr tan ( ) (6/3b)
are inclined from their normals in
the direction to oppose the motion.
Graphical solution. The accuracy of a graphical solution is well within the Also, we note the equal and opposite
uncertainty of the friction coefficients and provides a simple and direct result. By reactions R2 and R2.
laying off the vectors to a reasonable scale following the sequence described, we
obtain the magnitudes of P and the R’s easily by scaling them directly from the It should be evident that we avoid si-
diagrams. multaneous equations by eliminat-
ing reference to R3 for the block and
R1 for the wedge.

Article 6/5 Screws 359 358 Chapter 6 Friction

Force Analysis y Range of R2 y


Consider the square-threaded jack, Fig. 6/6, under the action of the α φ
2φ – α
φ R2
axial load W and a moment M applied about the axis of the screw. The
screw has a lead L (advancement per revolution) and a mean radius r. R2 φ –α φ φ –α
The force R exerted by the thread of the jack frame on a small represen-
tative portion of the screw thread is shown on the free-body diagram of
the screw. Similar reactions exist on all segments of the screw thread α α α
x
where contact occurs with the thread of the base.
If M is just sufficient to turn the screw, the thread of the screw φ
φ –α φ –α φ R1
will slide around and up on the fixed thread of the frame. The angle
made by R with the normal to the thread is the angle of friction, so that R1 2φ – α
tan . The moment of R about the vertical axis of the screw is Range of R1
Rr sin ( ), and the total moment due to all reactions on the threads
(a) Slipping impending (b) Range of R1 = R2 (c) Slipping impending
is ΣRr sin ( ). Since r sin ( ) appears in each term, we may fac- at upper surface for no slip at lower surface
tor it out. The moment equilibrium equation for the screw becomes
Figure 6/4
M [r sin ( )] ΣR

Equilibrium of forces in the axial direction further requires that with the case of raising the load. The free-body diagrams and vector
polygons for this condition are shown in Fig. 6/5.
W ΣR cos ( ) [cos ( )] ΣR Wedge problems lend themselves to graphical solutions as indicated
in the three figures. The accuracy of a graphical solution is easily held
Combining the expressions for M and W gives within tolerances consistent with the uncertainty of friction coefficients.
Algebraic solutions may also be obtained from the trigonometry of the
M Wr tan ( ) (6/3) equilibrium polygons.

To determine the helix angle , unwrap the thread of the screw for one
complete turn and note that tan 1 (L/2 r). R3
We may use the unwrapped thread of the screw as an alternative
model to simulate the action of the entire screw, as shown in Fig. mg
mg φ –α
6/7a. The equivalent force required to push the movable thread up the
R3 R2
fixed incline is P M/r, and the triangle of force vectors gives Eq. 6/3
immediately.
R2 φ
φ R2
φ –α
φ R1
W α P R2
W
R1
P
φ
M
M Forces to lower load

Figure 6/5

L
6/5 Screws
r α Screws are used for fastening and for transmitting power or motion.
R In each case the friction developed in the threads largely determines the
α action of the screw. For transmitting power or motion the square thread
φ
is more efficient than the V-thread, and the analysis here is confined to
Figure 6/6 the square thread.
Article 6/7 Thrust Bearings; Disk Friction 371 Article 6/8 Flexible Belts 377

SAMPLE PROBLEM 6 /8
T
6/8 Flexible Belts
r
The bell crank fits over a 100-mm-diameter shaft which is fixed and cannot The impending slippage of flexible cables, belts, and ropes over
rotate. The horizontal force T is applied to maintain equilibrium of the crank dθ
120 mm sheaves and drums is important in the design of belt drives of all types, β θ
under the action of the vertical force P 100 N. Determine the maximum and band brakes, and hoisting rigs.
minimum values which T may have without causing the crank to rotate in either Figure 6/11a shows a drum subjected to the two belt tensions T1
direction. The coefficient of static friction between the shaft and the bearing M
mm and T2, the torque M necessary to prevent rotation, and a bearing reac-
surface of the crank is 0.20. 100 O R
tion R. With M in the direction shown, T2 is greater than T1. The free-
body diagram of an element of the belt of length r d is shown in part b
of the figure. We analyze the forces acting on this differential element
Solution. Impending rotation occurs when the reaction R of the fixed shaft on 180 mm by establishing the equilibrium of the element, in a manner similar to T2
the bell crank makes an angle tan 1 with the normal to the bearing sur- P = 100 N that used for other variable-force problems. The tension increases from T1
face and is, therefore, tangent to the friction circle. Also, equilibrium requires T at the angle to T dT at the angle d . The normal force is a dif- (a)
that the three forces acting on the crank be concurrent at point C. These facts C ferential dN, since it acts on a differential element of area. Likewise the
T1
are shown in the free-body diagrams for the two cases of impending motion. friction force, which must act on the belt in a direction to oppose slip- n
The following calculations are needed:
ping, is a differential and is dN for impending motion.
T + dT
1 1 Equilibrium in the t-direction gives
Friction angle tan tan 0.20 11.31
t
Radius of friction circle rƒ r sin 50 sin 11.31 9.81 mm R1 µ dN T
r d d dN r
T cos dN (T dT) cos dθ dθ
120 2 2 –– ––
Angle tan 1
33.7 rƒ O 2 dθ 2
180
P = 100 N or dN dT
rƒ 9.81
1 1
Angle sin sin 2.60
OC (120)2 (180)2 R1 (b)
P since the cosine of a differential quantity is unity in the limit. Equilib-
– rium in the n-direction requires that Figure 6/11
T1
(a) Impending counterclockwise motion. The equilibrium triangle of forces (a) Counterclockwise motion impends d d
dN (T dT) sin T sin
is drawn and gives C
2 2
T2
T1 P cot ( ) 100 cot (33.7 2.60 ) or dN Td
T1 Tmax 165.8 N Ans.
where we have used the facts that the sine of a differential angle in the
r limit equals the angle and that the product of two differentials must be
O
neglected in the limit compared with the first-order differentials re-

(b) Impending clockwise motion. The equilibrium triangle of forces for this maining.
case gives R2
Combining the two equilibrium relations gives
P = 100 N
T2 P cot ( ) 100 cot (33.7 2.60 )
R2 dT
P d
T2 Tmin 136.2 N Ans. T
+
T2
Integrating between corresponding limits yields
(b) Clockwise motion impends
T2
dT
d
T

© Media Bakery
T1 0

T2
or ln
T1

where the ln (T2/T1) is a natural logarithm (base e). Solving for T2 gives Just one turn of a line around a
fixed cylinder can produce a large
T2 T1e (6/7) change in tension.

378 Chapter 6 Friction Article 6/9 Rolling Resistance 379

Note that is the total angle of belt contact and must be expressed in SAMPLE PROB LEM 6 /9
radians. If a rope were wrapped around a drum n times, the angle α
A flexible cable which supports the 100-kg load is passed over a fixed circu-
would be 2 n radians. Equation 6/7 holds equally well for a noncircular P
lar drum and subjected to a force P to maintain equilibrium. The coefficient of r
section where the total angle of contact is . This conclusion is evident static friction between the cable and the fixed drum is 0.30. (a) For 0, de- O
from the fact that the radius r of the circular drum in Fig. 6/11 does not termine the maximum and minimum values which P may have in order not to
enter into the equations for the equilibrium of the differential element raise or lower the load. (b) For P 500 N, determine the minimum value which
of the belt. the angle may have before the load begins to slip.
The relation expressed by Eq. 6/7 also applies to belt drives where
100 kg
both the belt and the pulley are rotating at constant speed. In this case
the equation describes the ratio of belt tensions for slippage or impend-
ing slippage. When the speed of rotation becomes large, the belt tends to Solution. Impending slipping of the cable over the fixed drum is given by Eq. P
6/7, which is T2/T1 e .
leave the rim, so Eq. 6/7 involves some error in this case.
β = π /2

6/9 Rolling Resistance O


(a) With 0 the angle of contact is /2 rad. For impending upward mo-
Deformation at the point of contact between a rolling wheel and its tion of the load, T2 Pmax, T1 981 N, and we have
supporting surface introduces a resistance to rolling, which we mention 981 N
only briefly. This resistance is not due to tangential friction forces and Pmax/981 e0.30( /2)
Pmax 981(1.602) 1572 N Ans. (a) α = 0
therefore is an entirely different phenomenon from that of dry friction.
α
To describe rolling resistance, we consider the wheel shown in Fig. For impending downward motion of the load, T2 981 N and T1 Pmin. Thus,
P
6/12 under the action of a load L on the axle and a force P applied at its β
r center to produce rolling. The deformation of the wheel and supporting 981/Pmin e0.30( /2)
Pmin 981/1.602 612 N Ans.
O
P surfaces as shown is greatly exaggerated. The distribution of pressure p
L over the area of contact is similar to the distribution shown. The resul-
tant R of this distribution acts at some point A and must pass through (b) With T2 981 N and T1 P 500 N, Eq. 6/7 gives us 981 N
a A
the wheel center for the wheel to be in equilibrium. We find the force P (b) P = 500 N
necessary to maintain rolling at constant speed by equating the mo- 981/500 e0.30 0.30 ln(981/500) 0.674
p ments of all forces about A to zero. This gives us
R 360
2.25 rad or 2.25 128.7 Helpful Hints
2
a
Figure 6/12 P rL rL We are careful to note that must
128.7 90 38.7 Ans.
be expressed in radians.

where the moment arm of P is taken to be r. The ratio r a/r is


In our derivation of Eq. 6/7 be cer-
called the coefficient of rolling resistance. This coefficient is the ratio of tain to note that T2 T1 .
resisting force to normal force and thus is analogous to the coefficient
of static or kinetic friction. On the other hand, there is no slippage or As was noted in the derivation of Eq.
impending slippage in the interpretation of r. 6/7, the radius of the drum does not
Because the dimension a depends on many factors which are diffi- enter into the calculations. It is only
cult to quantify, a comprehensive theory of rolling resistance is not the angle of contact and the coeffi-
available. The distance a is a function of the elastic and plastic proper- cient of friction which determine the
ties of the mating materials, the radius of the wheel, the speed of travel, limiting conditions for impending
and the roughness of the surfaces. Some tests indicate that a varies only motion of the flexible cable over the
curved surface.
slightly with wheel radius, and thus a is often taken to be independent
of the rolling radius. Unfortunately, the quantity a has also been called
the coefficient of rolling friction in some references. However, a has the
dimension of length and therefore is not a dimensionless coefficient in
the usual sense.
380 Chapter 6 Friction Article 7/3 Equilibrium 405

SAMPLE PROB LEM 6 /10 ms = 0.20 rolling wheel in Fig. 7/9b, on the other hand, does no work if the wheel
r r
does not slip as it rolls.
Determine the range of mass m over which the system is in static equilib- 9 kg
L/3 In Fig. 7/9c the moment Mƒ about the center of the pinned joint due
rium. The coefficient of static friction between the cord and the upper curved 30° d
ms = 0.40 to the friction forces which act at the contacting surfaces does negative
surface is 0.20, while that between the block and the incline is 0.40. Neglect fric- 2L/3
A work during any relative angular movement between the two parts.
tion at the pivot O. m
25° Thus, for a virtual displacement between the two parts, which have
O the separate virtual displacements 1 and 2 as shown, the negative
q = 40°
work done is Mƒ 1 Mƒ 2 Mƒ( 1 2), or simply Mƒ . For
Solution. From the FBD of the uniform slender bar, we can determine the TA each part, Mƒ is in the sense to oppose the relative motion of rotation.
tension TA in the cable at point A. It was noted earlier in the article that a major advantage of the
35°
2L L method of virtual work is in the analysis of an entire system of con-
[ΣMO 0] TA cos 35 9(9.81) cos 25 0
3 2 A G nected members without taking them apart. If there is appreciable ki-
TA 73.3 N netic friction internal to the system, it becomes necessary to dismember
O the system to determine the friction forces. In such cases the method of
25°
I. Motion of m impends up the incline. Ox virtual work finds only limited use.
The tension TA 73.3 N is the larger of the two tensions associated with
L/6 9(9.81) N
the rough rounded surface. From Eq. 6/7 we have Mechanical Efficiency
L/2
[T2 T1e s ] 73.3 T1e0.20[30 40 ] /180
T1 57.4 N Oy Because of energy loss due to friction, the output work of a machine
is always less than the input work. The ratio of the two amounts of
From the FBD of the block for Case I: work is the mechanical efficiency e. Thus,
mg
[ΣFy 0] N mg cos 40 0 N = 0.766mg T1 = 57.4 N output work
e
input work
[ΣFx 0] 57.4 mg sin 40 0.40(0.766mg) 0

mg 60.5 N m 6.16 kg y The mechanical efficiency of simple machines which have a single de-
gree of freedom and which operate in a uniform manner may be deter-
II. Motion of m impends down the incline. mined by the method of work by evaluating the numerator and mg
40° δs
The value TA 73.3 N is unchanged, but now this is the smaller of the two denominator of the expression for e during a virtual displacement.
tensions in Eq. 6/7. N x As an example, consider the block being moved up the inclined
0.40N
plane in Fig. 7/10. For the virtual displacement s shown, the output T
[T2 T1e s ] T2 73.3e0.20[30 40 ] /180
T2 93.5 N Case I work is that necessary to elevate the block, or mg s sin . The input µ k mg cos θ
θ
Considering the FBD of the block for Case II, we see that the normal force N is work is T s (mg sin kmg cos ) s. The efficiency of the inclined
unchanged from Case I. plane is, therefore, Figure 7/10
mg
[ΣFx 0] 93.5 – 0.4(0.766mg) mg sin 40 0 T2 = 93.5 N
mg s sin 1
e
mg 278 N m 28.3 kg mg(sin k cos ) s 1 k cot
0.40N
So the requested range is 6.16 m 28.3 kg. Ans. As a second example, consider the screw jack described in Art. 6/5
and shown in Fig. 6/6. Equation 6/3 gives the moment M required to
40° raise the load W, where the screw has a mean radius r and a helix angle
N x , and where the friction angle is tan 1 k. During a small rotation
of the screw, the input work is M Wr tan ( ). The output
Case II
work is that required to elevate the load, or Wr tan . Thus the effi-
Helpful Hints ciency of the jack can be expressed as
Only the total angular contact enters
Eq. 6/7 (as ). So our results are in- Wr tan tan
e
dependent of the quantities r and d. Wr tan ( ) tan ( )

Re-solve the entire problem if the As friction is decreased, becomes smaller, and the efficiency ap-
ramp angle were changed to 20 , proaches unity.
with all other given information re-
maining constant. Be alert for a sur-
prising result!

406 Chapter 7 Virtual Work Article 7/3 Equilibrium 407

SAMPLE PROB LEM 7 /1 SAMPLE PROB LEM 7 /2


P M
Each of the two uniform hinged bars has a mass m and a length l, and is The mass m is brought to an equilibrium position by the application of the
supported and loaded as shown. For a given force P determine the angle for couple M to the end of one of the two parallel links which are hinged as shown.
equilibrium. The links have negligible mass, and all friction is assumed to be absent. Deter-
l l mine the expression for the equilibrium angle assumed by the links with the
vertical for a given value of M. Consider the alternative of a solution by force and θ θ
θ b b
moment equilibrium.
Solution. The active-force diagram for the system composed of the two mem-
bers is shown separately and includes the weight mg of each bar in addition to
the force P. All other forces acting externally on the system are reactive forces G
+x m
which do no work during a virtual movement x and are therefore not shown. Solution. The active-force diagram shows the weight mg acting through the
The principle of virtual work requires that the total work of all external ac- P center of mass G and the couple M applied to the end of the link. There are no
tive forces be zero for any virtual displacement consistent with the constraints. other external active forces or moments which do work on the system during a M
Thus, for a movement x the virtual work becomes +h change in the angle .
The vertical position of the center of mass G is designated by the distance h
[ U 0] P x 2mg h 0 below the fixed horizontal reference line and is h b cos c. The work done
We now express each of these virtual displacements in terms of the variable , θ
by mg during a movement h in the direction of mg is
the required quantity. Hence, θ θ
mg h mg (b cos c) b b
mg mg +h
x 2l sin and x l cos mg( b sin 0) c
2 2
Helpful Hints mgb sin m G
Similarly,
Note carefully that with x positive to
The minus sign shows that the work is negative for a positive value of . The mg
l l the right x is also positive to the
h cos and h sin constant c drops out since its variation is zero.
2 2 4 2 right in the direction of P, so that the
With measured positive in the clockwise sense, is also positive clock-
virtual work is P( x). With h posi- Helpful Hint
Substitution into the equation of virtual work gives us wise. Thus, the work done by the clockwise couple M is M . Substitution into
tive down h is also mathematically
the virtual-work equation gives us Again, as in Sample Problem 7/1, we
l positive down in the direction of mg,
Pl cos 2mg sin 0 are consistent mathematically with
2 4 2 so that the correct mathematical ex- [ U 0] M mg h 0
our definition of work, and we see
pression for the work is mg( h).
from which we get which yields that the algebraic sign of the result-
When we express h in terms of in
ing expression agrees with the phys-
the next step, h will have a negative M mgb sin
2P 1 2P ical change.
tan mg or 2 tan mg Ans. sign, thus bringing our mathematical
2
expression into agreement with the 1 M
sin Ans.
To obtain this result by the principles of force and moment summation, it physical observation that the weight mgb
would be necessary to dismember the frame and take into account all forces act- mg does negative work as each center
Inasmuch as sin cannot exceed unity, we see that for equilibrium, M is limited
ing on each member. Solution by the method of virtual work involves a simpler of mass moves upward with an in-
to values that do not exceed mgb.
operation. crease in x and .
The advantage of the virtual-work solution for this problem is readily seen
when we observe what would be involved with a solution by force and moment
We obtain h and x with the same
equilibrium. For the latter approach, it would be necessary for us to draw sepa-
mathematical rules of differentiation
rate free-body diagrams of all of the three moving parts and account for all of the
with which we may obtain dh and dx.
internal reactions at the pin connections. To carry out these steps, it would be
necessary for us to include in the analysis the horizontal position of G with re-
spect to the attachment points of the two links, even though reference to this po-
sition would finally drop out of the equations when they were solved. We
conclude, then, that the virtual-work method in this problem deals directly with
cause and effect and avoids reference to irrelevant quantities.
408 Chapter 7 Virtual Work Article 7/4 Potential Energy and Stability 423

SAMPLE PROB LEM 7 /3 SAMPLE PROB LEM 7 /4


P
For link OA in the horizontal position shown, determine the force P on the B The 10-kg cylinder is suspended by the spring, which has a stiffness of 2
sliding collar which will prevent OA from rotating under the action of the couple h b y kN/m. Plot the potential energy V of the system and show that it is minimum at
M. Neglect the mass of the moving parts. M the equilibrium position. k = 2 kN/m
O
A
a x V=0
+x
Solution. The given sketch serves as the active-force diagram for the system. Solution. (Although the equilibrium position in this simple problem is clearly
–δ x
All other forces are either internal or nonworking reactive forces due to the con- where the force in the spring equals the weight mg, we will proceed as though
straints. this fact were unknown in order to illustrate the energy relationships in the sim-
We will give the crank OA a small clockwise angular movement as our B′ plest way.) We choose the datum plane for zero potential energy at the position
B m = 10 kg
virtual displacement and determine the resulting virtual work done by M and P. where the spring is unextended.
From the horizontal position of the crank, the angular movement gives a down- δθ The elastic potential energy for an arbitrary position x is Ve 21 kx2 and the
8
ward displacement of A equal to O a A gravitational potential energy is mgx, so that the total potential energy is
1 2
y a [V Ve Vg] V 2 kx mgx 6
A′
aδθ = δ y Ve = 1
– kx2
where is, of course, expressed in radians. Equilibrium occurs where 2
4
From the right triangle for which link AB is the constant hypotenuse we
dV dV V, J
may write Helpful Hints 0 kx mg 0 x mg/k
dx dx 2
b2 x2 y2 Note that the displacement a of 0.049
Although we know in this simple case that the equilibrium is stable, we x, m
point A would no longer equal y if 0
We now take the differential of the equation and get prove it by evaluating the sign of the second derivative of V at the equilibrium 0.02 0.04 0.06 0.08 0.10
the crank OA were not in a horizon-
position. Thus, d2V/dx2 k, which is positive, proving that the equilibrium is x = mg/k
y tal position. –2
0 2x x 2y y or x stable.
x y Substituting numerical values gives V = Ve + Vg
The length b is constant so that
Thus, 1
–4
b 0. Notice the negative sign, V (2000)x2 10(9.81)x
2 Vg = –mgx
which merely tells us that if one
y expressed in joules, and the equilibrium value of x is –6
x change is positive, the other must be
xa
negative.
x 10(9.81)/2000 0.0490 m or 49.0 mm Ans. –8
and the virtual-work equation becomes
We could just as well use a counter- We calculate V for various values of x and plot V versus x as shown. The
y clockwise virtual displacement for
[ U 0] M P x 0 M P minimum value of V occurs at x 0.0490 m where dV/dx 0 and d 2V/dx 2 is
xa 0
the crank, which would merely re-
Helpful Hints
positive.
Mx Mx verse the signs of all terms. The choice is arbitrary but simplifies
P Ans. the algebra.
ya ha

Again, we observe that the virtual-work method produces a direct relation- We could have chosen different
ship between the active force P and the couple M without involving other forces datum planes for Ve and Vg without
which are irrelevant to this relationship. Solution by the force and moment affecting our conclusions. Such a
equations of equilibrium, although fairly simple in this problem, would require change would merely shift the sepa-
accounting for all forces initially and then eliminating the irrelevant ones. rate curves for Ve and Vg up or down
but would not affect the position of
the minimum value of V.

424 Chapter 7 Virtual Work Article 7/4 Potential Energy and Stability 425

SAMPLE PROB LEM 7 /5 SAMPLE PROB LEM 7 /6 y


The two uniform links, each of mass m, are in the vertical plane and are The ends of the uniform bar of mass m slide freely in the horizontal and
Vg = 0 P
connected and constrained as shown. As the angle between the links increases O vertical guides. Examine the stability conditions for the positions of equilibrium.
C
with the application of the horizontal force P, the light rod, which is connected The spring of stiffness k is undeformed when x 0.
b
at A and passes through a pivoted collar at B, compresses the spring of stiffness k b
k. If the spring is uncompressed in the position where 0, determine the force A
P which will produce equilibrium at the angle . B b
Solution. The system consists of the spring and the bar. Since there are no ex- θ
b θ b
ternal active forces, the given sketch serves as the active-force diagram. We will
take the x-axis as the datum for zero gravitational potential energy. In the dis-
Solution. The given sketch serves as the active-force diagram of the system. placed position the elastic and gravitational potential energies are x
The compression x of the spring is the distance which A has moved away from B, 1 2 1 2 b k
Ve kx kb sin2 and Vg mg cos x
which is x 2b sin /2. Thus, the elastic potential energy of the spring is 2 2 2
2
1 2 1 The total potential energy is then Helpful Hints
[Ve 2
kx ] Ve 2
k 2b sin 2kb2 sin2
2 2 1 2 1
2
V Ve Vg 2 kb sin 2 mgb cos With no external active forces there
With the datum for zero gravitational potential energy taken through the is no U term, and V 0 is equiva-
support at O for convenience, the expression for Vg becomes Equilibrium occurs for dV/d 0 so that
lent to dV/d 0.
dV 1 1
[Vg mgh] Vg 2mg b cos kb2 sin cos 2 mgb sin (kb2 cos 2 mgb) sin 0
2 d

The distance between O and C is 4b sin /2, so that the virtual work done by The two solutions to this equation are given by
P is mg
sin 0 and cos Be careful not to overlook the solu-
2kb tion 0 given by sin 0.
U P 4b sin 2Pb cos
2 2 We now determine the stability by examining the sign of the second deriva-
The virtual-work equation now gives tive of V for each of the two equilibrium positions. The second derivative is
d2 V 1
[ U Ve Vg ] kb2(cos2 sin2 ) 2 mgb cos
d 2
1
2Pb cos 2kb2 sin2 2mgb cos kb2(2 cos2 1) 2 mgb cos
2 2 2

2kb2 sin cos mgb sin


2 2 2 Solution I. sin 0, 0

d2V 1 mg
Simplifying gives finally kb2(2 1) 2 mgb kb2 1
d 2 2kb
1
P kb sin 2 mg tan 2 Ans. positive (stable) if k mg/2b
2

If we had been asked to express the equilibrium value of corresponding to negative (unstable) if k mg/2b Ans.
a given force P, we would have difficulty solving explicitly for in this particular Thus, if the spring is sufficiently stiff, the bar will return to the vertical position
case. But for a numerical problem we could resort to a computer solution and even though there is no force in the spring at that position. We might not have anticipated this
graphical plot of numerical values of the sum of the two functions of to deter- result without the mathematical
mine the value of for which the sum equals P. analysis of the stability.
mg mg
Solution II. cos , cos 1
2kb 2kb

d2 V mg 2
1 mg mg 2
kb2 2 1 2 mgb 2kb kb2 1 Ans.
d 2 2kb 2kb Again, without the benefit of the
Since the cosine must be less than unity, we see that this solution is limited to mathematical analysis of the stability
the case where k mg/2b, which makes the second derivative of V negative. we might have supposed erroneously
Thus, equilibrium for Solution II is never stable. If k mg/2b, we no longer have that the bar could come to rest in a
Solution II since the spring will be too weak to maintain equilibrium at a value stable equilibrium position for some
of between 0 and 90 . value of between 0 and 90 .
446 Appendix A Area Moments of Inertia Article A/2 Definitions 447

SAMPLE PROB LEM A/1 y y0 SAMPLE PROB LEM A/3 y


Determine the moments of inertia of the rectangular area about the cen- Calculate the moments of inertia of the area of a circle about a diametral
troidal x0- and y0-axes, the centroidal polar axis z0 through C, the x-axis, and the dy axis and about the polar axis through the center. Specify the radii of gyration.
polar axis z through O. h dr0
–– r
2
y r0
x
x0 Solution. A differential element of area in the form of a circular ring may be O
Solution. For the calculation of the moment of inertia Ix about the x0-axis, a C used for the calculation of the moment of inertia about the polar z-axis through
horizontal strip of area b dy is chosen so that all elements of the strip have the h
–– O since all elements of the ring are equidistant from O. The elemental area is
same y-coordinate. Thus, 2 dA 2 r0 dr0, and thus,
h/2 O x r
1 r4 1
[Ix y2 dA] Ix y2b dy 12 bh
3
Ans. b [Iz r 2 dA] Iz r02(2 r0 dr0) 2 Ar
2
Ans.
h/2 0 2
y
By interchange of symbols, the moment of inertia about the centroidal y0-axis is The polar radius of gyration is dr0
1
Helpful Hint
Iy 3
12hb Ans.
k
I
kz
r
Ans. dθ
If we had started with the second- A
The centroidal polar moment of inertia is 2 r0
order element dA dx dy, integra-
r0 sinθ
1 1 tion with respect to x holding y By symmetry Ix Iy, so that from Eq. A/3 θ x
[Iz Ix Iy ] Iz 12
(bh3 hb3) 12
A(b2 h2) Ans. O
constant amounts simply to multi- r
plication by b and gives us the ex- 1 r4 1
By the parallel-axis theorem the moment of inertia about the x-axis is [Iz Ix Iy] Ix I
2 z 4
Ar 2 Ans.
pression y2b dy, which we chose at 4
2
1 h 1 3 1 the outset.
[Ix Ix Adx2] Ix 12 bh
3
bh 3 bh 3 Ah
2
Ans. The radius of gyration about the diametral axis is
2
I r
We also obtain the polar moment of inertia about O by the parallel-axis theorem, k kx Ans. Helpful Hints
A 2
which gives us
Polar coordinates are certainly indi-
2 2 The foregoing determination of Ix is the simplest possible. The result may
1 b h cated here. Also, as before, we
[Iz Iz Ad2] Iz 12 A(b
2
h2) A also be obtained by direct integration, using the element of area dA r0 dr0 d
2 2 choose the simplest and lowest-order
shown in the lower figure. By definition
1 element possible, which is the differ-
Iz 3
A(b2 h2) Ans. 2 r
ential ring. It should be evident im-
[Ix y2 dA] Ix (r0 sin )2r0 dr0 d mediately from the definition that
0 0
the polar moment of inertia of the
2
SAMPLE PROB LEM A/2 r4 sin2 ring is its area 2 r0 dr0 times r02.
d
x′ 0 4
Determine the moments of inertia of the triangular area about its base and 2 This integration is straightforward,
about parallel axes through its centroid and vertex. r4 1 sin 2 r4 but the use of Eq. A/3 along with the
Ans.
4 2 2 4
dy
0 result for Iz is certainly simpler.
h

Solution. A strip of area parallel to the base is selected as shown in the figure, x
y
and it has the area dA x dy [(h y)b/h] dy. By definition
x
h h b
h y y3 y4 bh3
[Ix y2 dA] Ix y2 b dy b Ans.
0 h 3 4h 0
12
Helpful Hints
By the parallel-axis theorem the moment of inertia I about an axis through the
centroid, a distance h/3 above the x-axis, is Here again we choose the simplest
possible element. If we had chosen
bh3 bh h 2
bh3 dA dx dy, we would have to inte-
[I I Ad2] I Ans.
12 2 3 36 grate y2 dx dy with respect to x first.
This gives us y2x dy, which is the ex-
A transfer from the centroidal axis to the x -axis through the vertex gives pression we chose at the outset.
bh3 bh 2h 2
bh3 Expressing x in terms of y should
[I I Ad2] Ix Ans. cause no difficulty if we observe the
36 2 3 4
proportional relationship between
the similar triangles.

448 Appendix A Area Moments of Inertia Article A/2 Definitions 449

SAMPLE PROB LEM A/4 y SAMPLE PROB LEM A/6 y


Determine the moment of inertia of the area under the parabola about the Calculate the moment of inertia about the x-axis of the area enclosed be- x
dx
3
x-axis. Solve by using (a) a horizontal strip of area and (b) a vertical strip of area. 2
tween the y-axis and the circular arcs of radius a whose centers are at O and A.
k y
x=

Solution. The constant k 49 is obtained first by substituting x 4 and y 3 Solution. The choice of a vertical differential strip of area permits one integra- a
into the equation for the parabola. 0 y2
x tion to cover the entire area. A horizontal strip would require two integrations
0 4
with respect to y by virtue of the discontinuity. The moment of inertia of the y1
2 /9
strip about the x-axis is that of a strip of height y2 minus that of a strip of height
(a) Horizontal strip. Since all parts of the horizontal strip are the same dis- y
4y y1. Thus, from the results of Sample Problem A/1 we write
tance from the x-axis, the moment of inertia of the strip about the x-axis is y2 dA x= dy A
x x
1 2 1 2 1 3 O a/2 a
where dA (4 x) dy 4(1 y2/9) dy. Integrating with respect to y gives us Solution (a) dIx 3 (y2 dx)y2 3 (y1 dx)y1 3 (y2 y1 3) dx
3 The values of y2 and y1 are obtained from the equations of the two curves,
y2 72 y
[Ix y2 dA] Ix 4y2 1 dy 14.4 (units)4 Ans. which are x2 y2 2 a2 and (x a)2 y12 a2, and which give y2 a2 x2
0 9 5
x and y1 a2 (x a)2. Thus, Helpful Hint
a/2
y 1 We choose the positive signs for the
y /9 {(a2 a)2} dx
2
(b) Vertical strip. Here all parts of the element are at different distances Ix x2) a2 x2 [a2 (x a)2] a2 (x
x =4 3 0 radicals here since both y1 and y2 lie
from the x-axis, so we must use the correct expressions for the moment of inertia
Solution (b) above the x-axis.
of the elemental rectangle about its base, which, from Sample Problem A/1, is Simultaneous solution of the two equations which define the two circles gives
y
bh3/3. For the width dx and the height y the expression becomes the x-coordinate of the intersection of the two curves, which, by inspection, is
1 a/2. Evaluation of the integrals gives
dIx 3
(dx)y3 x
x dx a/2
a4 3
To integrate with respect to x, we must express y in terms of x, which gives a2 a2 x2 dx
0 4 2 3
y 3 x/2, and the integral becomes Helpful Hint
a/2
a4 3
1
4
3 x 3
72 There is little preference between So- x2 a2 x2 dx
Ix dx 14.4 (units)4 Ans. 0 16 4 3
3
0 2 5 lutions (a) and (b). Solution (b) re-
a/2
quires knowing the moment of inertia a4 3 2
a2 a2 (x a)2 dx
for a rectangular area about its base. 0 4 2 3

SAMPLE PROB LEM A/5 a/2


a4 3
(x a)2 a2 (x a)2 dx
0 8 8 3
Find the moment of inertia about the x-axis of the semicircular area. C
x0
20 mm _
r Collection of the integrals with the factor of 13 gives
x′
a4
15 mm Ix (9 3 2 ) 0.0969a4 Ans.
Solution. The moment of inertia of the semicircular area about the x -axis is 96
one-half of that for a complete circle about the same axis. Thus, from the results x
of Sample Problem A/3, If we had started from a second-order element dA dx dy, we would write y2 dx dy
for the moment of inertia of the element about the x-axis. Integrating from y1 to y2
1 r4 204 holding x constant produces for the vertical strip
Ix 2 (104) mm4
2 4 8 y2
1
dIx y2 dy dx (y 3
3 2
y1 3) dx
We obtain the moment of inertia I about the parallel centroidal axis x0 next. y1
Transfer is made through the distance r 4r/(3 ) (4)(20)/(3 ) 80/(3 ) mm
by the parallel-axis theorem. Hence, which is the expression we started with by having the moment-of-inertia result
Helpful Hint
for a rectangle in mind.
2 This problem illustrates the caution
202 80
[I I Ad 2] I 2(10 4) 1.755(104) mm4
2 3 we should observe in using a double
transfer of axes since neither the x -
Finally, we transfer from the centroidal x0-axis to the x-axis. Thus, nor the x-axis passes through the
2 centroid C of the area. If the circle
202 80
[I I Ad 2] Ix 1.755(10 4) 15 were complete with the centroid on
2 3
the x axis, only one transfer would
1.755(104) 34.7(104) 36.4(104) mm4 Ans. be needed.
Article A/3 Composite Areas 457 458 Appendix A Area Moments of Inertia

SAMPLE PROBLEM A/7 y


SAMPLE PROB LEM A/8
30 mm
Determine the moments of inertia about the x- and y-axes for the shaded Calculate the moment of inertia and radius of gyration about the x-axis for 30 mm
area. Make direct use of the expressions given in Table D/3 for the centroidal the shaded area shown. Wherever possible, make expedient use of tabulated
moments of inertia of the constituent parts. 30 mm moments of inertia.
30 mm 30 mm
x
30 mm 40 mm 40 mm
Solution. The given area is subdivided into the three subareas shown—a rec- Solution. The composite area is composed of the positive area of the rectangle
tangular (1), a quarter-circular (2), and a triangular (3) area. Two of the sub- x (1) and the negative areas of the quarter circle (2) and triangle (3). For the rec-
areas are “holes” with negative areas. Centroidal x0 y0 axes are shown for areas 40 mm 40 mm tangle the moment of inertia about the x-axis, from Sample Problem A/1 (or
(2) and (3), and the locations of centroids C2 and C3 are from Table D/3. Table D/3), is x′ _
y r (2)
The following table will facilitate the calculations. 1 2 1 2 x0
y0 Ix 3 Ah 3 (80)(60)(60) 5.76(106) mm4 C
(1)
r From Sample Problem A/3 (or Table D/3), the moment of inertia of the negative
quarter-circular area about its base axis x is (3)
r x
(2)
x0 1 r4
C2 Ix (30) 4 0.1590(106) mm4
4 4 16
y0 Helpful Hints
(1)
We now transfer this result through the distance r 4r/(3 ) 4(30)/(3 )
12.73 mm by the transfer-of-axis theorem to get the centroidal moment of iner- Note that we must transfer the mo-
(3) ment of inertia for the quarter-
x0 tia of part (2) (or use Table D/3 directly).
C3 10 mm circular area to its centroidal axis x0
x
(30)2 before we can transfer it to the
r = 4(30)
–––– = 12.73 mm 40
–– [I I Ad2] Ix 0.1590(106) (12.73)2
3π 3 4 x-axis, as was done in Sample Prob-
mm lem A/5.
0.0445(106) mm4

The moment of inertia of the quarter-circular part about the x-axis is now We watch our signs carefully here.
Since the area is negative, both I
(30)2 and A carry negative signs.
[I I Ad2] Ix 0.0445(106) (60 12.73)2
4
Always use common sense at key
1.624(106) mm4
points such as this. The two minus
Finally, the moment of inertia of the negative triangular area (3) about its base, signs are consistent with the fact
from Sample Problem A/2 (or Table D/3), is that subareas (2) and (3) reduce the
1 1
numerical value of the moment of
Ix 12
bh3 12
(40)(30)3 0.90(106) mm4 inertia of the basic rectangular area.
The total moment of inertia about the x-axis of the composite area is,
consequently,

Ix 5.76(106) 1.624(106) 0.09(106) 4.05(106) mm4 Ans.


[Ix Σ Ix Σ Adx2] Ix 1.366(106) 2.68(106) 4.05(106) mm4 Ans.
This result agrees with that of Sample Problem A/7.
[Iy Σ Iy Σ Ady2] Iy 2.46(106) 4.90(106) 7.36(106) mm4 Ans.
The net area of the figure is A 60(80) 14 (30)2 1
2
(40)(30) 3490 mm2 so
The following sample problem will determine Ix by a different technique. For that the radius of gyration about the x-axis is
example, the area moment of inertia of subareas (1) and (3) about the x-axis are
kx Ix/A 4.05(106)/3490 34.0 mm Ans.
commonly tabulated quantities. While the above solution began with the centroidal
moments of inertia of subareas (1) and (3), the following sample problem will make
more direct use of the tabulated moments of inertia about the baselines.

468 Appendix A Area Moments of Inertia Article A/4 Products of Inertia and Rotation of Axes 469

SAMPLE PROB LEM A/9 y y0 SAMPLE PROB LEM A/12 y


10
Determine the product of inertia of the rectangular area with centroid at C dy Determine the orientation of the principal axes of inertia through the cen- mm 2.5 mm
with respect to the x-y axes parallel to its sides. troid of the angle section and determine the corresponding maximum and mini-
mum moments of inertia.

h x0
Solution. Since the product of inertia Ixy about the axes x0-y0 is zero by sym- C Solution. The location of the centroid C is easily calculated, and its position is
metry, the transfer-of-axis theorem gives us 50 mm
dx shown on the diagram.
[Ixy Ixy dxdyA] Ixy dxdybh Ans. C
b x
Products of Inertia. The product of inertia for each rectangle about its cen- 7.5 mm
In this example both dx and dy are shown positive. We must be careful to be con- x
troidal axes parallel to the x-y axes is zero by symmetry. Thus, the product of in-
sistent with the positive directions of dx and dy as defined, so that their proper ertia about the x-y axes for part I is 10 mm
signs are observed.
[Ixy Ixy dxdyA] Ixy 0 ( 12.5)( 7.5)(400) 3.75(104) mm4 40 mm
SAMPLE PROB LEM A/10 y where dx (7.5 5) 12.5 mm
y
Determine the product of inertia about the x-y axes for the area under the b and dy (20 10 2.5) 7.5 mm y′
parabola. 2
y Likewise for part II,
x =k II
dx [Ixy Ixy dxdyA] Ixy 0 (12.5)( 7.5)(400) 3.75(104) mm4 dy
x dy x′
Solution. With the substitution of x a when y b, the equation of the curve
y where dx (20 7.5) 12.5 mm, dy (5 2.5) 7.5 mm
becomes x ay2/b2.
a x
For the complete angle, dx α
C
y x
y0 b
Solution I. If we start with the second-order element dA dx dy, we have Ixy 3.75(104) 3.75(104) 7.5(104) mm4 dx
2
dIxy xy dx dy. The integral over the entire area is 2 /b
ay Moments of Inertia. The moments of inertia about the x- and y-axes for part I
b a b
a2y4 x=
1 2 1 2 2 y x0 I are
Ixy xy dx dy a y dy 6a b Ans.
0 ay2/b2 0 2 b4 dy
y/2 [I I Ad ] 2
Ix 1 3
(400)(12.5)2 6.58(104) mm4
12(40)(10)
a x 1
Iy (10)(40)3 (400)(7.5)2 7.58(104) mm4 Helpful Hint
Solution II. Alternatively we can start with a first-order elemental strip and x dx 12
save one integration by using the results of Sample Problem A/9. Taking a verti- and the moments of inertia for part II about these same axes are Mohr’s circle. Alternatively we could use Eqs.
y
cal strip dA y dx gives dIxy 0 (12 y)(x)(y dx), where the distances to the cen- a+x y0 1 A/11 to obtain the results for Imax and Imin, or
——– b [I I Ad2] Ix (10)(40)3 (400)(12.5)2 11.58(104) mm4
troidal axes of the elemental rectangle are dx y/2 and dy x. Now we have 2 dy 12 we could construct the Mohr’s circle from the
x 1 3
a a a Iy 12(40)(10) (400)(7.5)2 2.58(104) mm4 calculated values of Ix, Iy, and Ixy. These values
y2 xb2 b2 3 x0
Ixy x dx x dx x 1 2 2 are spotted on the diagram to locate points A
2 2a 6a 6a b Ans. Thus, for the entire section we have
0 0 0 and B, which are the extremities of the diame-
y
Ix 6.58(10)4 11.58(10)4 18.17(104) mm4 ter of the circle. The angle 2 and Imax and Imin
Helpful Hint
x Iy 7.58(104) 2.58(104) 10.17(104) mm4 are obtained from the figure, as shown.
a
If we had chosen a horizontal strip, our expression would have become
dI xy y 12 (a x)[(a x) dy], which when integrated, of course, gives us Principal Axes. The inclination of the principal axes of inertia is given by Eq. +Ixy , (10 4) mm4
the same result as before. A/10, so we have
Imax = 22.7
SAMPLE PROB LEM A/11 2Ixy 2( 7.50) Iy = 10.17
y tan 2 tan 2 1.875
Iy Ix 10.17 18.17
Determine the product of inertia of the semicircular area with respect to the y0 B
4r
–– –Ixy =
x-y axes. 3p 2 61.9 31.0 Ans.
x –(–7.5)
We now compute the principal moments of inertia from Eqs. A/9 using for = +7.5
r x0 I, (10 4) mm4
C and get Imax from Ix and Imin from Iy . Thus,
Solution. We use the transfer-of-axis theorem, Eq. A/8, to write 2α = 61.9°
18.17 10.17 18.17 10.17 Ixy = –7.5
4r r2 2r4 Imax (0.471) (7.50)(0.882) (104)
[Ixy Ixy dxdyA] Ixy 0 (r) Ans. 2 2 Imin =
3 2 3 Helpful Hint
22.7(104) mm4 Ans. 5.67 A
Proper use of the transfer-of-axis
where the x- and y-coordinates of the centroid C are dy r and dx 4r/(3 ).
theorem saves a great deal of labor 18.17 10.17 18.17 10.17 Ix = 18.17
Because y0 is an axis of symmetry, Ixy 0. Imin (0.471) (7.50)(0.882) (104)
in computing products of inertia. 2 2
4 4 –Ixy , (10 4) mm 4
5.67(10 ) mm Ans.
482 Appendix C Selected Topics of Mathematics 484 Appendix C Selected Topics of Mathematics

3. Miscellaneous relations 4. Law of sines With the aid of these identities and the distributive law, the vector
product may be written
sin2 cos2 1 a sin A
1 tan2 sec2 b sin B B P Q (Px i Py j Pz k) (Qx i Qy j Qz k)
1 cot2 csc2 c a (PyQz PzQy)i (PzQx PxQz)j (PxQy PyQx)k
1
sin 2 (1 cos )
2 The cross product may also be expressed by the determinant
1 A C D
cos 2
(1 cos )
2 b i j k
sin 2 2 sin cos P Q Px Py Pz
5. Law of cosines
cos 2 cos2 sin2 Qx Qy Qz
sin (a b) sin a cos b cos a sin b c2 a2 b2 2ab cos C
cos (a b) cos a cos b sin a sin b c2 a2 b2 2ab cos D 8. Additional relations
Triple scalar product (P Q) R R (P Q). The dot and cross
may be interchanged as long as the order of the vectors is main-
tained. Parentheses are unnecessary since P (Q R) is meaning-
less because a vector P cannot be crossed into a scalar Q R. Thus,
C/7 Vector Operations the expression may be written

1. Notation. Vector quantities are printed in boldface type, and P Q R P Q R


scalar quantities appear in lightface italic type. Thus, the vector
quantity V has a scalar magnitude V. In longhand work vector Q
P The triple scalar product has the determinant expansion
quantities
l
should always be consistently indicated by a symbol such
as V or V to distinguish them from scalar quantities.
Px Py Pz
R
P Q R Qx Qy Qz
Q Rx Ry Rz
2. Addition
Triangle addition P Q R P
Parallelogram addition P Q R Triple vector product (P Q) R R (P Q) R
R (Q P). Here we note that the parentheses must be used since an
Commutative law P Q Q P
Q expression P Q R would be ambiguous because it would not
Associative law P (Q R) (P Q) R identify the vector to be crossed. It may be shown that the triple
P vector product is equivalent to
P
3. Subtraction (P Q) R R PQ R QP

–Q
or P (Q R) P RQ P QR
P Q P ( Q) P– Q

The first term in the first expression, for example, is the dot prod-
uct R P , a scalar, multiplied by the vector Q.
4. Unit vectors i, j, k
9. Derivatives of vectors obey the same rules as they do for scalars.
V Vxi Vy j Vzk z
dP
where V V Vx 2 Vy 2 Vz 2 k Ṗ Ṗ x i Ṗy j Ṗz k
dt
kVz V
d(Pu)
Pu̇ Ṗu
y
dt
5. Direction cosines l, m, n are the cosines of the angles between V
and the x-, y-, z-axes. Thus, d(P Q)
jVy j P Q̇ Ṗ Q
dt
l Vx /V m Vy /V n Vz /V
d(P Q)
so that V V(li mj nk) iVx i
P Q̇ Ṗ Q
dt
and l2 m2 n2 1 x

Article C/9 Derivatives 485 486 Appendix C Selected Topics of Mathematics

10. Integration of vectors. If V is a function of x, y, and z and an el- C/10 Integrals


ement of volume is d dx dy dz, the integral of V over the volume
may be written as the vector sum of the three integrals of its com-
ponents. Thus, xn 1
xn dx
n 1
Vd i Vx d j Vy d k Vz d
dx
ln x
x

C/8 Series 2
a bx dx (a bx)3
(Expression in brackets following series indicates range of 3b
convergence.)
n(n 1) 2 n(n 1)(n 2) 3 2
(1 x)n 1 nx x x [x2 1] x a bx dx (3bx 2a) (a bx)3
2! 3! 15b2
x3 x5 x7 2
sin x x [x ] 2
3! 5! 7! x2 a bx dx (8a2 12abx 15b2x2) (a bx)3
105b3
x2 x4 x6 2
cos x 1 [x ]
2! 4! 6!
dx 2 a bx
ex e x x3 x5 x7 a bx b
sinh x x [x2 ]
2 3! 5! 7!
a x a x
ex e x x2 x4 x6 dx a x b x (a b) sin 1
cosh x 1 [x2 ] a b
2 2! 4! 6! b x
a0 n x n x x dx
ƒ(x) an cos bn sin 1 [a bx a ln (a bx)]
2 n 1 l n 1 l a bx b2
l l
1 n x 1 n x
where an ƒ(x) cos dx, bn ƒ(x) sin dx (a bx)1 n
l l l l l l x dx a bx a
(a bx)n b2 2 n 1 n
[Fourier expansion for l x l]
dx 1 1
x ab 1 1
x ab
tan a or tanh a
a bx2 ab ab

C/9 Derivatives x dx 1
ln (a bx2)
a bx2 2b
u du dv
d v v u
dxn d(uv) 1
u dv v du , dx dx x2 a2 dx 2 a2 a2 ln (x x2 a2)]
nxn 2[x x
1,
dx dx dx dx dx v2
1 x
lim sin x sin dx tan dx dx a2 x2 dx 2
x a2 x2 a2 sin 1
a
xl0

1
lim cos x cos dx 1 x a2 x2 dx 3 (a
2
x2)3
xl0

d sin x d cos x d tan x x a2 x


cos x, sin x, sec2 x x2 a2 x2 dx (a2 x2)3 x a2 x2 a2 sin 1
dx dx dx 4 8 a
d sinh x d cosh x d tanh x
cosh x, sinh x, sech2 x x3 a2 x2 dx
1 2 2 2 2
x2)3
dx dx dx 5(x 3 a ) (a
Article C/10 Integrals 487 488 Appendix C Selected Topics of Mathematics

dx 1 b 1 b 2cx eax
ln a bx cx2 x c or sin 1 eax dx
a
a bx cx2 c 2 c c b2 4ac
eax
dx xeax dx (ax 1)
ln (x x2 a2) a2
x2 a2
eax (a sin px p cos px)
dx x eax sin px dx
sin 1
a a2 p2
a2 x2
eax (a cos px p sin px)
x dx eax cos px dx
x2 a2 a2 p2
x2 a2

x dx eax 2
a2 x2 eax sin2 x dx a sin2 x sin 2x a
a2 x2 4 a2

x x2 a2 dx 1 2 a2)3 eax 2
3 (x eax cos2 x dx a cos2 x sin 2x a
4 a2

x a2 a4 eax a
x2 x2 a2 dx (x2 a2)3 x x2 a2 ln (x x2 a2) eax sin x cos x dx sin 2x cos 2x
4 8 8 4 a2 2

sin x dx cos x cos x


sin3 x dx (2 sin2 x)
3

sin x
cos x dx sin x cos 3 x dx (2 cos2 x)
3

1 ln 1 sin x 2 1
sec x dx cos 5 x dx sin x 3
sin3 x 5
sin5 x
2 1 sin x

x sin 2x x sin x dx sin x x cos x


sin2 x dx
2 4

x sin 2x x cos x dx cos x x sin x


cos2 x dx
2 4

x2 sin x dx 2x sin x (x2 2) cos x


sin2 x
sin x cos x dx
2
2 2
x cos x dx 2x cos x (x 2) sin x
sinh x dx cosh x

cosh x dx sinh x

tanh x dx ln cosh x

ln x dx x ln x x

Useful Tables 497 498 Appendix D Useful Tables

TABLE D/3 PROPERTIES OF PLANE FIGURES TABLE D/3 PROPERTIES OF PLANE FIGURES Continued

AREA MOMENTS AREA MOMENTS


FIGURE CENTROID FIGURE CENTROID
OF INERTIA OF INERTIA

Rectangular Area bh3


Ix = –––
r y0 3
α – C – r sin α
Arc Segment α r r = –––––– —
α
– bh3
C Ix = –––
h x0 — 12
x – bh
Quarter and Semicircular Arcs Iz = –– (b2 + h2)
b 12

C C – 2r
– y = –– —
y r
bh3
a x1 – a+b Ix = –––
x = –––– 12
y 3

x C h –
Triangular Area bh3
y –
y Ix = –––
36
r4 x – h
Ix = Iy = ––– y = ––
4 b 3 bh3
r — Ix1 = –––
Circular Area x 4
C r4
Iz = –––
2

y
Area of Elliptical
Quadrant –
x = 4a
––
ab3 I– = –– – 4
Ix = ––––,
16
x
16 9 (
–– ab3
)
3
y
8
r4
Ix = Iy = –––

x –
y = 4b
––
a3b I– = –– – 4
Iy = ––––,
16
y
16 9 (
–– a3b
)
b C

( )
– 8 – 3
– 4r Ix = – – –– r4 y ab
Semicircular
C y = –– x Iz = ––– (a2 + b2)
3 8 9 a 16
Area r –
y
x r4
Iz = –––
4 Subparabolic Area
ab3
Ix = –––
b – 21
x = 3a
2 — 2
r4 y y = kx = a2 x ––
4
y Ix = Iy = ––– a3b
16 Iy = –––
Area A = ab
–– 5
3 –
( ) C b
– – – x –
y = 3b
x = – 4r
y = –– 4
Ix = Iy = — – –– r4 ––

( )
– 3 –
y 10 a3 b2
Quarter-Circular x C 16 9 Iz = ab –– + ––
Area r x 5 21
– a
y
x r4
Iz = –––
8
Parabolic Area
2ab3
Ix = ––––
y –
y r4 (α – 1 sin 2α )
Ix = –– –
b
y = kx2 = — x2 x = 3a
–– 7
4 2 a2 8
a
r 2a3b
Iy = ––––

Area of Circular α – x

x =2 r sin α
– –––––– r4 (α + 1 sin 2 )
Iy = –– – α Area A = 2ab
––– b
x C –
15
Sector α x 3 α 4 2 3 – y = 3b
––
C
( )
y 5 a2 b2
Iz = 2ab –– + ––
Iz = 1
– r4α x 15 7
2

You might also like